TP Ap
TP Ap
Linear Programming: II
Transportation Model
4.1. INTRODUCTION
In operations Research Linear programming is one of the model in mathematical programming, which
is very broad and vast. Mathematical programming includes many more optimization models known as
Non - linear Programming, Stochastic programming, Integer Programming and Dynamic Programming
- each one of them is an efficient optimization technique to solve the problem with a specific structure,
which depends on the assumptions made in formulating the model. We can remember that the general
linear programming model is based on the assumptions:
(a) Certainty
The resources available and the requirement of resources by competing candidates, the profit
coefficients of each variable are assumed to remain unchanged and they are certain in nature.
(b) Linearity
The objective function and structural constraints are assumed to be linear.
(c) Divisibility
All variables are assumed to be continuous; hence they can assume integer or fractional values.
(e) Non-negativity
A non-negativity constraint exists in the problem, so that the values of all variables are to be ≥ 0,
i.e. the lower limit is zero and the upper limit may be any positive number.
For the data given above, the mathematical model will be:
Minimize Z = c11 x11 + c12 x12 + c13 x13 + c14 x14 + c21 x21 + c22 x22 + c23 x23 + c24 x24 +
c31 x31 + c32 x32 + c33 x33 + c34 x34 subject to a condition: OBJECTIVE FUNCTION.
a11 x11 +a12 x12 + a13 x13 + a14 x14 ≤ b1 (because the sum must be less than or equal to the
available capacity)
a21 x21 + a22 x22 + a23 x23 + a24 x24 ≤ b2
a31 x31 + a32 x32 + a33 x33 + a34 x34 ≤ b3 MIXED STRUCTURAL CONSTRAINTS.
Minimize Z = ∑ ∑c
j =1 i =1
ij x j s.t. where i = 1 to m and j = 1 to n.
∑a
i =1
ij xij ≤ bi where i = 1 to m and j = 1 to n
∑a
j =1
ij x ji ≥ d j where i = 1 to m and j = 1 to n
Similarities
1. Both have objective function.
2. Both have linear objective function.
3. Both have non - negativity constraints.
4. Both can be solved by simplex method. In transportation model it is laborious.
5. A general linear programming problem can be reduced to a transportation problem if (a) the
aij's (coefficients of the structural variables in the constraints) are restricted to the values 0
and/or 1 and (b) There exists homogeneity of units among the constraints.
Differences
1. Transportation model is basically a minimization model; where as general linear programming
model may be of maximization type or minimization type.
144 Operations Research
2. The resources, for which, the structural constraints are built up is homogeneous in
transportation model; where as in general linear programming model they are different. That
is one of the constraint may relate to machine hours and next one may relate to man-hours
etc. In transportation problem, all the constraints are related to one particular resource or
commodity, which is manufactured by the factories and demanded by the market points.
3. The transportation problem is solved by transportation algorithm; where as the general linear
programming problem is solved by simplex method.
4. The values of structural coefficients (i.e. xij) are not restricted to any value in general linear
programming model, where as it is restricted to values either 0 or 1 in transportation problem.
Say for example:
Let one of the constraints in general linear programming model is: 2x –3y +10z ≤ 20. Here
the coefficients of structural variables x, y and z may negative numbers or positive numbers
of zeros. Where as in transportation model, say for example x11 + x12 + x13 + x14 = bi = 20.
Suppose the value of variables x11, and x14 are 10 each, then 10 + 0. x12 + 0. x13 + 10 = 20.
Hence the coefficients of x11 and x14 are 1 and that of x12 and x13 are zero.
A 4 3 2 10
B 5 6 1 8
C 6 4 3 5
D 3 5 4 6
Here Σb is greater than Σd hence we have to open a dummy column whose requirement constraint
is 6, so that total of availability will be equal to the total demand. Now let get the basic feasible solution
by three different methods and see the advantages and disadvantages of these methods. After this let us
give optimality test for the obtained basic feasible solutions.
For cell AX the availability constraint is 10 and the requirement constraint is 7. Hence 7 is smaller
than 10, allocate 7 to cell AX. Next 10 – 7 = 3, this is allocated to cell AY to satisfy availability
requirement. Proceed in the same way to complete the allocations. Then count the allocations, if it is
equals to m + n – 1, then the solution is basic feasible solution. The solution, we got have 7 allocations
which is = 4 + 4 – 1 = 7. Hence the solution is basic feasible solution.
Linear Programming: II Transportation Model 147
A X 7 7 × 4 = 28
A Y 3 3 × 3 = 09
B Y 8 8 × 6 = 48
C Y 1 1 × 4 = 04
C Z 4 4 × 3 = 12
D Z 1 1 × 4 = 04
D DUMMY 5 5 × 0 = 00
4.4.4. Solution by Least cost cell (or inspection) Method: (Matrix Minimum
method)
(i) Identify the lowest cost cell in the given matrix. In this particular example it is = 0. Four cells
of dummy column are having zero. When more than one cell has the same cost, then both the cells are
competing for allocation. This situation in transportation problem is known as tie. To break the tie,
select any one cell of your choice for allocation. Make allocations to this cell either to satisfy availability
constraint or requirement constraint. Once one of these is satisfied, then mark crosses (×) in all the
cells in the row or column which ever has completely allocated. Next search for lowest cost cell. In
the given problem it is cell BZ which is having cost of Re.1/- Make allocations for this cell in similar
manner and mark crosses to the cells in row or column which has allocated completely. Proceed this
way until all allocations are made. Then write allocations and find the cost of transportation. As the
total number of allocations are 7 which is equals to 4 + 4 – 1 = 7, the solution is basic feasible solution.
148 Operations Research
(Note: The numbers under and side of rim requirements shows the sequence of allocation and the units
remaining after allocation)
Allocations are:
A Y 8 8 × 3 = 24
A Z 2 2 × 2 = 04
B Z 3 3 × 1 = 03
B DUMMY 5 5 × 0 = 00
C X 1 1 × 6 = 06
C Y 4 4 × 4 = 16
D X 6 6 × 3 = 18
Total in Rs. 71
through the cell having cost Rs.3/-, the cost of transportation per unit will be Rs. 3/-.
Instead we transport through the cell having cost of Rs.6/-, then the cost of transportation
will be Rs.6/- per unit. That is for not taking correct decision; we are spending Rs.3/- more
(Rs.6 – Rs.3 = Rs.3/-). This is the penalty for not taking correct decision and hence the
opportunity cost. This is the lowest opportunity cost in that particular row or column as
we are deducting the smallest element form the next highest element.
Note: If the smallest element is three and the row or column having one more three,
then we have to take next highest element as three and not any other element. Then
the opportunity cost will be zero. In general, if the row has two elements of the same
magnitude as the smallest element then the opportunity cost of that row or column is
zero.
(ii) Write row opportunity costs and column opportunity costs as described above.
(iii) Identify the highest opportunity cost among all the opportunity costs and write a tick (√ )
mark at that element.
(iv) If there are two or more of the opportunity costs which of same magnitude, then select any
one of them, to break the tie. While doing so, see that both availability constraint and
requirement constraint are simultaneously satisfied. If this happens, we may not get basic
feasible solution i.e solution with m + n – 1 allocations. As far as possible see that both are
not satisfied simultaneously. In case if inevitable, proceed with allocations. We may not get
a solution with, m + n – 1 allocations. For this we can allocate a small element epsilon (∈) to
any one of the empty cells. This situation in transportation problem is known as degeneracy.
(This will be discussed once again when we discuss about optimal solution).
In transportation matrix, all the cells, which have allocation, are known as loaded cells and
those, which have no allocation, are known as empty cells.
(Note: All the allocations shown in matrix 1 to 6 are tabulated in the matrix given below:)
150 Operations Research
Consider matrix (1), showing cost of transportation and availability and requirement constraints.
In the first row of the matrix, the lowest cost element is 0, for the cell A-Dummy and next highest
element is 2, for the cell AZ. The difference is 2 – 0 = 2. The meaning of this is, if we transport the load
through the cell A-Dummy, whose cost element is 0, the cost of transportation will be = Rs.0/- for
Linear Programming: II Transportation Model 151
each unit transported. Instead, if we transport the load through the cell, AZ whose cost element is Rs.
2/- the transportation cost is = Rs.2/- for each unit we transport. This means to say if we take decision
to send the goods through the cell AZ, whose cost element is Rs.2/- then the management is going to
loose Rs. 2/- for every unit it transport through AZ. Suppose, if the management decide to send load
through the cell AX, Whose cost element is Rs.4/-, then the penalty or the opportunity cost is Rs.4/-.
We write the minimum opportunity cost of the row outside the matrix. Here it is shown in brackets.
Similarly, we find the column opportunity costs for each column and write at the bottom of each
corresponding row (in brackets). After writing all the opportunity costs, then we select the highest
among them. In the given matrix it is Rs.3/- for the rows D and C. This situation is known as tie.
When tie exists, select any of the rows of your choice. At present, let us select the row D. Now in that
row select the lowest cost cell for allocation. This is because; our objective is to minimize the
transportation cost. For the problem, it is D-dummy, whose cost is zero. For this cell examine what
is available and what is required? Availability is 6 tons and requirement is 5 tons. Hence allocate 5 tons
to this cell and cancel the dummy row from the problem. Now the matrix is reduced to 3 × 4. Continue
the above procedure and for every allocation the matrix goes on reducing, finally we get all allocations
are over. Once the allocations are over, count them, if there are m + n – 1 allocations, then the solution
is basic feasible solution. Otherwise, the degeneracy occurs in the problem. To solve degeneracy, we
have to add epsilon (∈), a small element to one of the empty cells. This we shall discuss, when we
come to discuss optimal solution. Now for the problem the allocations are:
A X 3 3 × 4 = 12
A Y 7 7 × 3 = 21
B X 3 3 × 5 = 15
B Z 5 5 × 1 = 05
C Y 5 5 × 4 = 20
D X 1 1 × 3 = 03
D DUMMY 5 5 × 0 = 00
Total Rs. 76
Now let us compare the three methods of getting basic feasible solution:
152 Operations Research
In the problem given, the total cost of transportation for Northwest corner method is Rs. 101/-.
The total cost of transportation for Inspection method is Rs. 71/- and that of VAM is Rs. 76/-. The total
cost got by inspection method appears to be less. That of Northwest coroner method is highest. The
cost got by VAM is in between.
Now let us discuss the method of getting optimal solution or methods of giving optimality test for
basic feasible solution.
2. Starting from that cell draw a loop moving horizontally and vertically from loaded cell to
loaded cell. Remember, there should not be any diagonal movement. We have to take turn
only at loaded cells and move to vertically downward or upward or horizontally to reach
another loaded cell. In between, if we have a loaded cell, where we cannot take a turn,
ignore that and proceed to next loaded cell in that row or column.
3. After completing the loop, mark minus (–) and plus (+) signs alternatively.
4. Identify the lowest load in the cells marked with negative sign.
5. This number is to be added to the cells where plus sign is marked and subtract from the load
of the cell where negative sign is marked.
6. Do not alter the loaded cells, which are not in the loop.
7. The process of adding and subtracting at each turn or corner is necessary to see that rim
requirements are satisfied.
8. Construct a table of empty cells and work out the cost change for a shift of load from loaded
cell to loaded cell.
9. If the cost change is positive, it means that if we include the evaluated cell in the programme,
the cost will increase. If the cost change is negative, the total cost will decrease, by including
the evaluated cell in the programme.
10. The negative of cost change is the opportunity cost. Hence, in the optimal solution of
transportation problem empty cells should not have positive opportunity cost.
11. Once all the empty cells have negative opportunity cost, the solution is said to be optimal.
One of the drawbacks of stepping stone method is that we have to write a loop for every empty
cell. Hence it is tedious and time consuming. Hence, for optimality test we use MODI method rather
than the stepping stone method.
Let us take the basic feasible solution we got by Vogel's Approximation method and give optimality
test to it by stepping stone method.
Basic Feasible Solution obtained by VAM:
154 Operations Research
Table showing the cost change and opportunity costs of empty cells:
Table I.
1. AZ +AZ – AX + BX – BZ +2 – 4 + 5 – 1 = + 2 –2
2 A Dummy + A DUMMY – AX + BX – B DUMMY +0 – 4 + 3 – 0 = – 1 +1
3 BY + BY – AY + AX – BX +6 – 3 + 4 – 5 = +2 -2
4 B DUMMY + B DUMMY – BX + DX – D DUMMY +0 – 5 +3 – 0 = –2 +2
5 CX +CX – CY + AX – AY 6 – 4 + 3 – 4 = +1 –1
6 CZ +CZ – BZ + BX –AX + AY – CY +2 –1 +5 – 4 +5 – 4 =+1 –1
7 C DUMMY + C DUMMY – D DUMMY + DX – + 0 – 0 +3 – 4 +3 – 4 = +2
AX + AY – CY –2
8 DY +DY – DX + AX – AY +5 – 3 +4 – 3 = +3 –3
9 DZ +DZ – DX +BX – BZ +4 – 3 + 5 – 1 = +5 –5
In the table 1 cells A DUMMY, B DUMMY, C DUMMY are the cells which are having positive
opportunity cost. Between these two cells B DUMMY and C DUMMY are the cells, which are having
higher opportunity cost i.e Rs. 2/ - each. Let us select any one of them to include in the improvement
of the present programme. Let us select C DUMMY.
Linear Programming: II Transportation Model 155
Table II.
9 DZ + DZ – DX + BX – BZ + 4 – 3 +5 – 1 = + 5 –5
Cells A DUMMY and B DUMMY are having positive opportunity costs. The cell B DUMMY is
having higher opportunity cost. Hence let us include this cell in the next programme to improve the
solution.
Table III.
S.No. Empty Evaluation Opportunity
Cell Loop formation Cost change in Rs. Cost
1 AX +AX – AY + CY – C DUMMY + +4 – 3 + 4 – 0 + 0 – 5 = 0 0
B DUMMY – BX
2 AZ + AZ – BZ + B DUMMY – C DUMMY +2 –1 + 0 – 0 + 4 – 3 =+ 2 –2
+ CX – AX
3 A DUMMY + A DUMMY – C DUMMY + CY – AY +0 – 0 + 4 – 3 = +1 –1
4 BY + BY – B DUMMY + C DUMMY – CY +6 – 0 + 0 – 4 = + 2 –2
5 CX + CX – BX + B DUMMY – C DUMMY +6 – 5 + 0 – 0 = +1 –1
6 CZ + CZ – BZ + B DUMMY – C DUMMY +2 – 1 + 0 – 0 = +1 –1
7 DY +DY – CY + C DUMMY – B DUMMY +5 – 4 + 0 – 0 + 5 – 3 = +3 –3
+ BX – DX
8 DZ + DZ – BZ + BX – DX +4 – 1 + 5 – 3 = +5 –5
All the empty cells have negative opportunity cost hence the solution is optimal. The allocations
are:
1 AY 10 10 × 3 = 30
2 BX 01 01 × 5 = 05
3 BZ 05 05 × 1 = 05
4 B DUMMY 02 02 × 0 = 00
5 CY 02 02 × 4 = 08
6 C DUMMY 03 03 × 0 = 00
7 DX 06 06 × 3 = 18
Total in Rs. 66
Optimal allocation.
getting the opportunity cost of all the cells, we have to select the cell with highest positive opportunity
cost for including it in the modified solution.
Steps in MODI method:
1. Select row element (ui) and Column element (vj) for each row and column, such that ui + vj
= the actual cost of loaded cell. In MODI method we can evaluate empty cells simultaneously
and get the opportunity cost of the cell by using the formula (ui + vj) – Cij, where Cij is the
actual cost of the cell.
2. In resource allocation problem (maximization or minimization method), we have seen that
once any variable becomes basis variable, i.e., the variable enters the programme; its
opportunity cost or net evaluation will be zero. Here, in transportation problem also, once
any cell is loaded, its opportunity cost will be zero. Now the opportunity cost is given by (ui
+ vj) – Cij, which is, equals to zero for a loaded cell.
i.e. (ui + vj) – Cij = 0 which means, (ui + vj) = Cij. Here (ui + vj) is known as implied cost
of the cell. For any loaded cell the implied cost is equals to actual cost of the cell as its
opportunity cost is zero. For any empty cell, (implied cost – actual cost) will give
opportunity cost.
3. How to select ui and vj? The answer is:
(a) Write arbitrarily any one of them against a row or against a column. The written ui or
vj may be any whole number i.e ui or vj may be ≤ or ≥ to zero. By using the formula
(ui + vj) = Cij for a loaded cell, we can write the other row or column element. For
example, if the actual cost of the cell Cij = 5 and arbitrarily we have selected ui = 0,
then vj is given by ui + vj = 0 + vj = 5. Hence vj = –5. Like this, we can go from loaded
cell to loaded cell and complete entering of all ui s and vj s.
(b) Once we get all ui s and vj s, we can evaluate empty cells by using the formula (ui + vj)
– Actual cost of the cell = opportunity cost of the cell, and write the opportunity cost
of each empty cell at left hand bottom corner.
(c) Once the opportunity costs of all empty cells are negative, the solution is
said to be optimal. In case any cell is having the positive opportunity cost,
the programme is to be modified.
Remember the formula that IMPLIED COST OF A CELL = ui + vj
Opportunity cost of loaded cell is zero i.e (ui + vj) = Actual cost of the cell.
Opportunity cost of an empty cell = implied cost – actual cost of the cell = (ui
+ vj) – Cij
(d) In case of degeneracy, i.e. in a basic feasible solution, if the number of loaded
cells are not equals to m + n – 1, then we have to add a small element epsilon
∈), to any empty cell to make the number of loaded cells equals to m + n –
(∈
1. While adding '∈ ∈' we must be careful enough to see that this ∈ should not
form a closed loop when we draw horizontal and vertical lines from loaded
cell to loaded cell. In case the cell to which we have added ∈ forms a closed
loop, then if we cannot write all ui s and vj s.
∈ is such a small element such that a + ∈ = a or a – ∈ = a and ∈ – ∈ = 0.
158 Operations Research
Now let us take the basic feasible solution obtained by VAM method and apply MODI method of
optimality test.
Basic feasible solution got by VAM method.
The cell C DUMMY is having a positive opportunity cost. Hence we have to include this cell in the
programme. The solution has m + n – 1 allocations.
Linear Programming: II Transportation Model 159
The cell B DUMMY is having a positive opportunity cost. Thïs is to be included in the modified
programme.
As the opportunity cost of all empty cells are negative, the solution is optimal. The solution has m
+ n – 1 allocations.
The allocations are:
S.No Loaded Cell Load Cost in Rs.
1 AY 10 10 × 3 = 30
2. BX 01 01 × 5 = 05
3. BZ 05 05 × 1 = 05
4. B DUMMY 02 02 × 0 = 00
5. CY 02 02 × 4 = 08
6. C DUMMY 03 03 × 0 = 00
7. CX 06 06 × 3 = 18
Total Cost in Rs. 66
Readers can verify the optimal solution got by Stepping stone method and the MODI method they
are same. And they can also verify the opportunity costs of empty cells they are also same. This is the
advantage of using MODI method to give optimality test. Hence the combination of VAM and MODI
can be conveniently used to solve the transportation problem when optimal solution is asked.
2. By including the cell having zero as the opportunity cost, derive one more optimal solution,
let it be the matrix B.
3. The new matrix C is obtained by the formula: C = d A + (1– d) B, where 'd' is a positive
fraction less than 1.
It is better to take always d = 1/2, so that C = 1/2 A + 1/2 B.
Now we shall take the optimal solution of the problem above and write the alternate optimal
solutions.
Matrix A (First optimal Solution).
The cell AX, having zero opportunity cost is included in revised solution. The loop is:
+ AX – BX + B DUMMY – C DUMMY + CY – AY = + 4 – 5 + 0 – 0 + 4 – 3 = 0
Allocation:
(i) The given matrix is to be multiplied by –1, so that the problem becomes maximization problem.
Or ii) Subtract all the elements in the matrix from the highest element in the matrix. Then the problem
becomes maximization problem. Then onwards follow all the steps of maximization problem to get the
solution. Let us consider the same problem solved above.
Problem 4.2. Four factories, A, B, C and D produce sugar and the capacity of each factory is given
below: Factory A produces 10 tons of sugar and B produces 8 tons of sugar, C produces 5 tons of
sugar and that of D is 6 tons of sugar. The sugar has demand in three markets X, Y and Z. The demand
of market X is 7 tons, that of market Y is 12 tons and the demand of market Z is 4 tons. The following
matrix gives the returns the factory can get, by selling the sugar in each market. Formulate a transportation
problem and solve for maximizing the returns.
Profit in Rs. per ton (× 100) Availability in tons.
Markets.
X Y Z
Factories.
A 4 3 2 10
B 5 6 1 8
C 6 4 3 5
D 3 5 4 6
Requirement in tons. 7 12 4 Σ b = 29, Σ d = 23
Here Σ b is greater than Σ d hence we have to open a dummy column whose requirement
constraint is 6, so that total of availability will be equal to the total demand. Now let get the basic
feasible solution by VAM and then give optimality test by MODI method. The balanced matrix of the
transportation problem is:
By multiplying the matrix by –1, we can convert it into a maximisation problem. Now in VAM we
have to find the row opportunity cost and column opportunity costs. In minimisation problem, we use
to subtract the smallest element in the row from next highest element in that row for finding row
opportunity cost. Similarly, we use to subtract smallest element in the column by next highest element
Linear Programming: II Transportation Model 163
in that column to get column opportunity cost. Here as we have multiplied the matrix by –1 the highest
element will become lowest element. Hence subtract the lowest element from the next highest element
as usual. Otherwise, instead of multiplying by –1 simply find the difference between highest element
and the next lowest element and take it as opportunity cost of that row or column. For example in the
given problem in the row A, the highest element is 4 and the next lowest element is 3 and hence the
opportunity cost is 4 – 3 = 1. (Or smallest element is –4 and the next highest element is –3 and the
opportunity cost is –3 – (– 4) = –3 + 4 = 1). Similarly, we can write all opportunity costs. Once we find
the opportunity costs, rest of the procedure is same. That is, we have to select highest opportunity
cost and select the highest profit element in that row or column for allocation. Obtain the basic feasible
solution. As usual the basic feasible solution must have m + n – 1 allocations. If the allocations are not
equal to m + n – 1, the problem degenerate. In that case, add ∈ to an empty cell, which do not form
loop with other loaded cells. Once we have basic feasible solution, the optimality test by MODI method,
is followed. Here, once the opportunity costs of all the cells are positive, (as we have converted the
maximistion problem into minimisation problem) the solution is said to be optimal.
In the given problem as the opportunity costs of all empty cells are positive, the solution is
optimal. And the optimal return to the company is Rs. 125/-.
Allocations:
(1)
164 Operations Research
(2)
(3)
(4)
(5)
Linear Programming: II Transportation Model 165
The smallest load ∈ is added to cell XB which does not make loop with other loaded cells.
Linear Programming: II Transportation Model 167
The basic feasible solution is having four loaded cells. As the number of columns is 3 and number
of rows is 2 the total number of allocations must be 2 + 3 – 1 = 4. The solution got has four allocations.
Hence the basic feasible solution. Now let us give optimality test by MODI method.
Row numbers ui s and column numbers vj s are written in the matrix and opportunity cost of
empty cells are evaluated. As the opportunity cost of all empty cells are negative, the solution is
optimal. The allocations and the total cost of transportation is:
Let us make initial assignment by using Northwest corner method. To modify the solution we
include the cell O1D3 in the programme, as it is having highest opportunity cost.
Improved solution:
Linear Programming: II Transportation Model 169
Total number of allocations are less than m + n – 1. Hence we have to add one epsilon to an
empty cell. Remember, in transportation problem, which has minimization of cost as its objective
function a, we have to add epsilon to recently vacated cell, which is having lowest shipping cost. We
have a tie between two cells, i.e. O1D2 and O2D3. Let us select O1D2 to add epsilon. To improve the
solution, let us take empty cell O4D1 in the programme.
Improved Programme: The solution is not having m + n – 1 allocations. We have to add epsilon;
in the programme epsilon is added to cell 04D4
Revised Programme.
The epsilon is shifted to an empty cell. The improved solution is having 8 allocations. Hence a
feasible solution.
As the cell O1D4 having positive opportunity cost, let us include and revise the programme.
Revised programme. Cell 03D5 having positive opportunity cost is included in revised programme.
170 Operations Research
Revised programme: Cell O3D1 having positive opportunity cost is included in the revised
programme.
Revised Programme.
Linear Programming: II Transportation Model 171
As the opportunity costs of all empty cells are negative, the solution is optimal. The allocations
and the total cost of transportation is:
The same problem, if we solve by VAM, the very first allocation will be feasible and optimality
test shows that the solution is optimal.
Roc: Row opportunity cost, COC= Column opportunity cost, Avail: Availability, Req: Requirement.
D1 D2 D3 D4 D5 Avail ROC
O1 4 3 1 15 2 6 40 1
O2 5 2 3 4 5 30 1
O3 3 5 6 3 2 20 1
O4 2 4 4 5 3 10 1
REQ 30 30 15 20 5 100
COC 1 1 2 1 1
D1 D2 D4 D5 Avail ROC
O1 4 3 2 6 25 1
O2 5 2 30 4 5 30 2
O3 3 5 3 20 1
O4 2 4 5 3 10 1
REQ 30 30 20 5 85
COC 1 1 1 1
D1 D4 D 5 Avail ROC
O1 4 2 20 6 25 2
O3 3 3 2 20 1
O4 2 5 3 10 1
REQ 30 20 5 55
ROC 1 1 1
172 Operations Research
D1 D5 Avail ROC
O1 4 5 6 5 2
O3 3 2 20 1
O4 2 3 10 1
REQ 30 5 35
COC 1 1
D1 D5 Avail ROC
O3 3 2 20 1
O4 2 10 3 10 1
REQ 25 5 30
COC 1 1
D1 D5 AVAIL
O3 3 15 2 5 20
REQ 15 5 20
Allocation by VAM:
Allocations are same as in the optimal solution got by northwest corner method. All opportunity
costs of empty cells are negative. Hence the total transportation cost is Rs. 210/-
Linear Programming: II Transportation Model 173
1. Initial assignment by Northwest corner method: The Maximum time of allocated cell is 17
hours. Any cell having time element greater than 17 hours is cancelled, so that it will not in the programme.
174 Operations Research
By drawing loops, let us try to avoid 17 hours cell and include a cell, which is having time element
less than 17 hours. The basic feasible solution is having m + n – 1 allocations.
In this allocation highest time element is 11 hours. Let us try to reduce the same.
Linear Programming: II Transportation Model 175
In this allocation also the maximum time element is 11 hours. Let us try to avoid this cell.
No more reduction of time is possible. Hence the solution is optimal and the time required for
completing the transportation is 10 Hours. Tmax = 10 hours.
Problem. 4.7 M/S Epsilon traders purchase a certain type of product from three manufacturing units
in different places and sell the same to five market segments. The cost of purchasing and the cost of
transport from the traders place to market centers in Rs. per 100 units is given below:
Market Segments.
(Transportation cost in Rs.per 100 units).
Place of Availability Manufacturing 1 2 3 4 5
Manufacture. In units x 10000. cost in Rs. per unit
Bangalore (B) 10 40 40 30 20 25 35
Chennai (C) 15 50 30 50 70 25 40
Hyderabad (H) 5 30 50 30 60 55 40
Requirement in units × 10000 6 6 8 8 4
The trader wants to decide which manufacturer should be asked to supply how many to which
market segment so that the total cost of transportation and purchase is minimized.
Solution
Here availability is 300000 units and the total requirement is 320000 units. Hence a dummy row
(D) is to be opened. The following matrix shows the cost of transportation and purchase per unit in Rs.
from manufacturer to the market centers directly.
1 2 3 4 5 Availability
B 4040 4030 4020 4025 4035 10
C 5030 5050 5070 5025 5040 15
H 3050 3030 3060 3055 3040 5
D 0 0 0 0 0 2
Requirement. 6 6 8 8 4 32
176 Operations Research
Let us multiply the matrix by 100 to avoid decimal numbers and get the basic feasible solution by VAM.
Table. Avail: Availability. Req: Requirement, Roc: Row opportunity cost, Coc: Column opportunity
cost.
Tableau. I Cost of transportation and purchase Market segments.
Allocation:
From To Load Cost in Rs.
Bangalore 2 10,000 4,03,000
Bangalore 3 80,000 32, 16,000
Bangalore 5 10,000 4, 03,000
Chennai 1 60,000 30, 18,000
Chennai 4 80,000 40, 20,000
Chennai 5 10,000 5, 04,000
Hyderabad 2 50,000 15, 15,000
Total cost in Rs. 1,30, 79,000
Linear Programming: II Transportation Model 179
(a) If the sales price per unit at all warehouses is Rs. 70/- what would be the allocation for
maximum profit? Is it necessary to obtain a new solution or the solution obtained above
holds valid?
(b) If the sales prices are Rs.70/-, Rs. 80/-, Rs. 72/-, Rs. 68/- and Rs. 65/- at A, B, C, D and E
respectively what should be the allocation for maximum profit?
Solution: Total production including the overtime production is 1750 units and the total requirement
by warehouses is 1500 units. Hence the problem is unbalanced. This can be balance by opening a
Dummy Row (DR), with cost coefficients equal to zero and the requirement of units is 250. The cost
coefficients of all other cells are got by adding production and transportation costs. The production
cum transportation matrix is given below:
A B C D E DC Availability
X 52 54 58 53 56 0 300
Y 41 46 45 41 42 0 400
Z 56 57 59 56 54 0 600
XOT 57 59 63 58 61 0 100
YOT 50 55 54 50 51 0 150
ZOT 64 65 67 64 62 0 200
Requirement: 400 400 200 200 300 250 1750
180 Operations Research
As we have m +n – 1 (= 11) allocations, the solution is feasible and all the opportunity costs of
empty cells are negative, the solution is optimal.
Allocations:
Cell Load Cost in Rs.
XA 300 300 × 52 = 15,600
YD 100 100 × 41 = 4,100
YE 300 300 × 40 = 12,000
ZB 400 400 × 54 = 21,000
ZC 100 100 × 59 = 5,900
ZD 100 100 × 56 = 5,600
XOT A 50 50 × 57 = 2,850
XOT DR 50 50 × 0 = 0
YOT A 50 50 × 50 = 5,500
YOT C 100 100 × 54 = 5,400
ZOT DR 50 50 × 0 = 0
Total Cost in Rs. 75, 550
Linear Programming: II Transportation Model 181
Allocation by VAM:
(1)
A B C D E DC AVAIL ROC
X 52 54 58 53 56 0 300 52
Y 41 46 45 41 42 0 400 41
Z 56 57 59 56 54 0 600 54
XOT 57 59 63 58 61 0 100 50
YOT 50 55 54 50 51 0 150 50
ZOT 64 65 67 64 62 0 (200) 2 00 62
REQ 400 400 2 00 2 00 300 2 50 1750
COC 9 8 9 9 9 0
As for one allocation a row and column are getting eliminated. Hence, the degeneracy occurs.
(2) A B C D E DC AVAIL ROC
X 52 54 58 53 56 0 300 52
Y 41 46 45 41 42 0 400 41
Z 56 57 59 56 54 0 600 54
XOT 57 59 63 58 61 0 (50) 100 57
YOT 50 55 54 50 51 0 150 50
REQ 400 400 2 00 2 00 300 2 50 15 50
COC 9 8 9 9 9 0
(3)
A B C D E AVAIL ROC
X 52 54 58 53 56 300 1
Y 41 46 45 41 42 (300) 400 0
Z 56 57 59 56 54 600 2
XOT 57 59 63 58 61 50 2
YOT 50 55 54 50 51 150 0
REQ 400 400 2 00 2 00 300 1500
COC 9 8 9 9 9
Here also for one allocation, a row and a column are getting eliminated. Degeneracy will occur. In
all we may have to allocate two ∈ s to two empty cells.
182 Operations Research
(4)
A B C D AVAIL ROC
X 52 54 58 53 300 1
Y 41 46 45 41 (100) 100 0
Z 56 57 59 56 600 0
XOT 57 59 63 58 50 1
YOT 50 55 54 50 150 0
REQ 400 400 2 00 2 00 1200
COC 9 8 9 9
(5)
A B C D Avail Roc
X 52 54 58 53 300 1
Z 56 57 59 56 600 0
XOT 57 59 63 58 50 1
YOT 50 55 54 (150) 50 150 0
Req 400 400 200 100 1100
Coc 2 1 4 3
(6)
A B C D Avail Roc
X 52 (300) 54 58 53 300 1
Z 56 57 59 56 600 0
XOT 57 59 63 58 50 1
Req 400 400 50 100 950
Coc 4 3 1 3
(6)
A B C D Avail Roc
Z 56 57 59 (50) 56 550 0
XOT 57 59 63 58 50 1
Req 100 400 50 100 600
Coc 1 2 4 2
Linear Programming: II Transportation Model 183
(7)
A B D Avail Roc
Z 56 57 56 (100) 550 0
XOT 57 59 58 50 1
Req 100 400 100 600
Coc 1 2 2
(8)
A B Avail Roc
Z 56 57 (400) 450 1
XOT 57 59 50 2
Req 100 400 500
Coc 1 2
(9)
A Avail
Z 56 (50) 50
XOT 57 (50) 50
100
In the table showing optimal solution, we can understand that the company X has to work 50%
of its over time capacity, and company Y has to work 100% of its overtime capacity and company Z
will not utilize its overtime capacity.
(a) Here the total profit or return that the trading company gets is equals to Sales revenue – total
expenses, which include manufacturing cost and transportation cost. Hence,
Profit = (Total Sales Revenue) – (Manufacturing cost + transportation cost).
In the question given the sales price is same in all market segments, hence, the profit calculated is
independent of sales price. Hence the programme, which minimizes the total cost will, maximizes the
total profit. Hence the same solution will hold good. We need not work a separate schedule for
maximization of profit.
(b) Here sales price in market segments will differ. Hence we have to calculate the total profit by
the formula given above for all the markets and work for solution to maximise the profit.
184 Operations Research
As all the opportunity cost of empty cells are positive (maximization problem), the solution is
optimal.
The allocations are:
Cell Load Cost in Rs.
XB 300 300 × 26 = 7, 800
YA 400 400 × 29 = 11, 600
ZC 200 200 × 13 = 2, 600
ZD 50 50 × 12 = 600
ZE 300 300 × 11 = 3, 300
Z DR 50 50 × 0 = 0
XOT B 100 100 × 21 = 2, 100
YOT D 150 150 × 18 = 2, 700
ZOT DR 200 200 × 0 = 0
Profit in Rs. = 30, 700
Linear Programming: II Transportation Model 185
○
A B C D E DC Avail Coc
○
○
X 18 26 14 15 9 0 300 8
○
○
Y 29 34 27 27 25 0 400 5
○
○ ○ ○ ○ ○ ○ ○ ○ ○ ○ ○ ○ ○ ○ ○ ○ ○ ○ ○ ○ ○ ○ ○ ○ ○ ○ ○ ○ ○ ○ ○ ○ ○ ○ ○ ○ ○ ○ ○ ○ ○ ○ ○
○
400 ○
○
Z 14 23 13 12 11 0 600 9
○
○
XOT 13 21 9 10 4 0 100 8
○
YOT 20 25 18 18 14 0 150 5
○
○
ZOT 6 15 5 4 3 0 200 9
○
○
Coc 11 8 9 9 9 0
○
○
○
As for one allocation a row and column are getting eliminated. Hence, the degeneracy occurs.
(2)
B C D E DC Avail Coc
○ ○ ○ ○
X ○ ○ ○ ○ ○
26
○ ○ ○ ○ ○
14
○ ○ ○ ○ ○
15 ○ ○ ○ ○ ○
9 ○ ○ ○ ○
0 ○ ○ ○ ○ ○
300 ○ ○ ○ ○ ○
11
○ ○ ○ ○ ○
300
Z 23 13 12 11 0 600 10
XOT 21 9 10 4 0 100 11
YOT 25 18 18 14 0 150 7
ZOT 15 5 4 3 0 200 10
Req 400 200 200 300 250 1350
Coc 1 4 3 5 0
(3)
○
B C D E DC Avail Coc
○
○
Z 23 13 12 11 0 600 10
○
○
XOT 21 9 10 4 0 100 11
○
○ ○ ○ ○ ○ ○ ○ ○ ○ ○ ○ ○ ○ ○ ○ ○ ○ ○ ○ ○ ○ ○ ○ ○ ○ ○ ○ ○ ○ ○ ○ ○ ○ ○ ○ ○ ○ ○ ○ ○ ○ ○ ○
○
100
○
○
YOT 25 18 18 14 0 150 7
○
○
ZOT 15 5 4 3 0 200 10
○
○
Coc 2 5 6 3 0
○
○
○
Here also for one allocation, a row and a column are getting eliminated. Degeneracy will occur. In
all we may have to allocate two ∈ s to two empty cells.
186 Operations Research
○
(4)
○
C D E DC Avail Coc
○
Z 13 12 11 0 600 1
○
○
YOT 18 18 14 0 150 0
○
○
150
○
○
ZOT 5 4 3 0 200 1
○
○
Req 200 200 300 250 950
○
○
Coc 5 6 3 0
○
○
○
(5)
○
○
○
C D E DC Avail Coc
○
○
Z 13 12 11 0 600 1
○
200
○
○
ZOT 5 4 3 0 200 1
○
○
Coc 8 8 8 0
○
○
○
(6)
○
○
○
D E DC Avail Coc
○
○
Z 12 11 0 400 1
○
○
300
○
○
ZOT 4 3 0 200 1
○
○
Coc 8 8 0
○
○
○
(7)
(8)
Linear Programming: II Transportation Model 187
Problem. 4.9. A company has booked the orders for its consignment for the months of April, May,
June and July as given below:
April: 900 units, May: 800 units, June: 900 units and July: 600 units. The company can produce 750
units per month in regular shift, at a cost of Rs. 80/- per unit and can produce 300 units per month by
overtime production at a cost of Rs. 100/- per unit. Decide how much the company has to produce in
which shift for minimizing the cost of production. It is given that there is no holding cost of inventory.
Solution: Remember here the production of April is available to meet the orders of April and subsequent
months. But the production of May cannot be available to meet the demand of April. Similarly, the
production of June is not available to meet the demand of April, May, but it can meet the demand of
June and subsequent months and so on. Hence very high cost of production is allocated to the cells
(Infinity or any highest number greater than the costs given in the problem), which cannot meet the
demands of previous months (i.e. back ordering is not allowed). Here total availability is 4200 units and
the total demand is for 3200 units. Hence we have to open a dummy column (DC), with cost coefficients
equal to zero. The balanced matrix is shown below. Let us find the initial basic feasible solution by
Northwest corner method and apply optimality test by MODI method.
A: April, M: May, J: June, Jl: July, AT: April Over time, MT: May overtime, JT: June overtime, JLT:
July over time. DC : Dummy column.
Tableau 1.
Here the cell JT DC is having highest opportunity cost. Hence let us include the cell in the revised
programme. To find the opportunity costs of empty cells, the row number ui and column number vj are
188 Operations Research
shown. The cells marked with (X) are avoided from the programme. We can also allocate very high
cost for these cells, so that they will not enter into the programme.
Tableau II. Revised programme.
As the opportunity costs of all empty cells are either zeros or negative elements, the solution is
optimal. As many empty cells are having zero as the opportunity cost, they can be included in the
solution and get alternate solution.
Allocations:
Demand month. Production of the month Load Cost in Rs.
April April regular 750 750 × 80 = 60,000
April April over time 150 150 × 100 = 15,000
May April over time 150 150 × 100 = 15,000
May May regular 650 650 × 80 = 52,000
June May regular 100 100 × 80 = 8,000
June May over time 300 300 × 100 = 30,000
June June Regular 500 500 × 80 = 40,000
July June regular 250 250 × 80 = 20,000
July July regular 350 350 × 80 = 28,000
Dummy column June over time 300 300 × 0
Dummy Column July regular 300 300 × 0
Dummy column July over time 300 300 × 0
Total cost in Rs.: 2,68,000
Linear Programming: II Transportation Model 189
Problem: 4.10. Let us slightly change the details given in the problem 4.9. It is given that production
of a month could be stored and delivered in next month without extra costs. Let us now consider that
there is a cost associated with inventory holding or inventory carrying cost. Let the inventory carrying
cost is Rs. 20 per month decide the new allocation.
Solution: In the cost matrix, for regular production, the cost is Rs. 80/-, for overtime production, the
cost is Rs. 100 and for the stock held the inventory carrying cost is Rs. 20/ per month. If the stock is
held for two months the inventory carrying cost is Rs. 40/-. That is if the production of April is
supplied in June the cost will be Rs. 80/- + Rs. 40/- =
Rs. 120/- and do on. The initial basic feasible solution is obtained by Northwest corner method.
Cell AOT DC is having highest positive opportunity cost. Hence we have to include this in the
revised programme.
190 Operations Research
In the above matrix, two cells, MO M and JO J are having positive opportunity costs = 20.
Hence, they may be included in the revised programme. If we include them in the programme, the final
optimal solution will be as follows:
Linear Programming: II Transportation Model 191
As all the opportunity costs of empty cells are negative, the solution is optimal. The optimal
allocations are:
Month of demand Month of production load cost in Rs. Rs.
April April regular 750 750 × 80 = 60,000
April April over time 150 150 × 100 = 15,000
Dummy Col April over time 150 150 × 0 = 0
May May regular 750 750 × 80 = 60,000
May May over time 50 50 × 100 = 5,000
Dummy Column May over time 250 250 × 0 = 0
June June regular 750 750 × 80 = 60,000
June June over time 150 150 × 100 = 15,000
Dummy column June over time 150 150 × 0 = 0
July July regular 600 600 × 80 = 48,000
Dummy column July regular 150 150 × 0 = 0
Dummy column July overtime 300 300 × 0 = 0
Total cost in Rs. 2,63,000
192 Operations Research
Solution:
To formulate a transportation problem for three factories and four market centers, we have to
find out the cost coefficients of cells. For this, if we want the cost of the cell XA, the cost of transportation
from X to warehouse W1 + Cost transportation from W1 to market center A are calculated and as our
objective is to minimize the cost, the least of the above should be entered as the cost coefficient of cell
XA. Similarly, we have to workout the costs and enter in the respective cells.
Linear Programming: II Transportation Model 193
Cell XA: Route X-W1-A and X- W2- A minimum of these two (28 and 18) i.e 18
Cell XB Route X - W1 - B and X - W2 - B Minimum of the two is (29, 17) i.e 17
Cell XC Route X - W1 - C and X- W2 - C Minimum of the two is (27, 11) i.e 11
Cell XD Route X- W1 - D and X- W2 - D Minimum of the two is (34, 22) i.e 22
Similarly we can calculate for other cells and enter in the matrix. The required transportation
problem is:
A B C D Available
X 18 17 21 22 150
Y 18 17 21 22 100
Z 18 19 17 24 100
Required. 80 100 70 100 350
As the opportunity costs of all empty cells are negative, the solution is optimal. The optimal
allocation is:
Cell Route Load Cost in Rs. Rs.
XA X-W2-A 50 50 × 18 = 900 (The answer shows that the
XB X - W2 - B 100 100 × 17 = 1700 capacity of W2 is 250 units and
YB Y- W2 - B ε --- = ---- capacity of W1 is100 units).
YD Y - W2- D 100 100 × 22 = 2200
ZA Z- W1- A 30 30 × 18 = 540
ZC Z - W1 - C 70 70 × 17 = 1190
Total Cost in Rs. 6530
(1)
A B C D Avail Roc
X 18 17 21 22 150 0
Y 18 17 21 22 100 0
Z 18 19 17 24 100 2
70
Req. 80 100 70 100 350
Coc 18 17 15 22
(2)
A B D Avail Roc
X 18 17 22 150 1
Y 80 18 17 22 100 1
Z 18 19 24 30 1
Req. 80 100 100 280
Coc 0 0 0
(3)
B D Avail Roc
X 17 22 70 5
70
Y 17 22 100 5
Z 19 24 30 5
Req. 100 100 200
Coc 0 0
Linear Programming: II Transportation Model 195
(4)
B D Avail Roc
Y 17 22 100 5
30
Z 19 24 30 5
Req. 30 100 200
Coc 2 2
D Avail Roc
Y 22 70
70
Z 24 30
30
Req. 100 100
Coc
Problem 4.12.
2. W2 has no capacity limitation. However, it deals partial direct distribution of 80 units. Therefore,
as a source its availability should be the difference between the total availability from all
factories i.e X, Y and Z less its own direct distribution. 430 – 80 = 350.
3. As an intermediate destination, it should have the capacity to route entire production i.e. 430
units.
4. Unit cost of transportation from X, Y, and Z to destinations A, B, C and D, through W1 and
W2 can be had from figure given, this can be entered in the table- 1 showing the initial
transportation matrix.
5. There is no direct transportation from X,Y, and Z to destinations A, B, C and D. To avoid this
direct routes we can allocate very high cost of transportation costs for these cells or we can
avoid these cells by crossing them, i.e. eliminating them from the programme.
6. W1 as source giving to W1 as warehouse or sink, and W2 as a source giving to W2 as
warehouse or sink will have zero cost.
As the total number of allocations are m + n – 1 after allocating ∈ to cell W1A, the solution is a
basic feasible solution. By giving the optimality test by MODI method, we see that all the opportunity
costs of empty cells are negative and hence the solution is optimal.
The allocation:
Cell Load Cost in Rs. Rs.
XW2 150 150 × 19 = 1500
YW2 160 160 × 10 = 1600
ZW1 70 70 × 10 = 700
ZW2 50 50 × 12 = 600
W1A ε ----- -- -----
W 1C 70 70 × 7 = 140
W2 W 2 70 70 × 0 = 0
W 2A 80 80 × 8 = 160
W 2B 100 100 × 7 = 700
W2D 100 100 × 12 = 1200
Total Cost in Rs. 6,600
Linear Programming: II Transportation Model 197
VAM:
(1) W1 W2 A B C D Avail ROC
X 20 10 X X X X 150 10
Y 15 10 X X X X 160 5
Z 10 12 X X X X 120 2
70
W1 0 X 8 9 7 14 70 1
W2 X 0 8 7 11 12 350 1
Req. 70 430 80 100 70 100 850
COC 10 10 0 2 4 2
(2)
W2 A B C D Avail ROC
X 10 X X X X 150 10
Y 10 X X X X 160 10
Z 12 X X X X 50 12
50
W1 X 8 9 7 14 70 1
W2 0 8 7 11 12 350 7
Req. 430 80 100 70 100 780
COC 10 0 2 4 2
(3)
W2 A B C D Avail ROC
X 10 X X X X 150 10
150
Y 10 X X X X 160 10
W1 X 8 9 7 14 70 1
W2 0 8 7 11 12 350 7
Req. 380 80 100 70 100 730
COC 10 0 2 4 2
198 Operations Research
(4)
W2 A B C D Avail ROC
Y 10 X X X X 160 10
160
W1 X 8 9 7 14 70 1
W2 0 8 7 11 12 350 7
Req. 230 80 100 70 100 580
COC 10 0 2 4 2
(5)
W2 A B C D Avail ROC
W1 X 8 9 7 14 70 1
W2 0 8 7 11 12 350 7
70
Req. 70 80 100 70 100 420
COC INF 0 2 4 2
(6)
A B C D Avail ROC
W1 8 9 7 14 70 1
70
W2 8 7 11 12 280 1
Req. 80 100 70 100 350
COC 0 2 4 2
(7)
A B D Avail ROC
W2 8 7 12 280
80 100 100
Req. 80 100 100 280
COC
as far as mathematical principles are concerned. But for a transportation manager, it enables him to plan
for more than one orthogonal path for an or several cells to evaluate penalty costs, which obviously will
be different for different paths.
In the solution shown above as all the opportunity costs of empty cells are negative. Consider
empty cell XA. Its opportunity cost is Rs. -3/- This means to say that the units cost of transportation
of cell XA decreases by Rs.3/- or more i.e Rs.10/- the unit cost of transportation of the empty cell XA
minus 3 = 7, or less than 7 the optimal solution changes, i.e. the cell XA will become eligible for
entering into solution. Hence this cost, which shows the limit of the unit cost of empty cell, is known
as implied cost in transportation problems. We can see that the opportunity cost of empty cell ZE is
zero. This shows that the cell ZE is as good as a loaded cell and hence we can write alternate solutions
by taking the cell ZE into consideration. (Note: No unit cost of transportation is given for the cell
ZC. Hence that cell should not be included in the programme. For this purpose, we can cross
the cell or allocate very high unit cost of transportation for the cell. In case zero or any
negative element is given as the unit cost of transportation for a cell, the value can be taken
for further treatment.)
200 Operations Research
Cells XA and XB is positive when θ is > than 19. Cell XC is positive when θ is > 17 and cell XD
is positive when θ is > 27. Other cells are not influenced by θ .
If unit cost of transportation increase and becomes 17, the present optimum may change. In
case the unit cost of transportation of the cell XA is reduced, the solution will still remain optimum, as
our objective is to minimize the total transportation cost.
A point to note here is we have used Northwest corner method and Vogel’s approximation method
to get basic feasible solution. Also we have discussed the least cost method and there are some methods
such as row minimum and column minimum methods. These methods attempt to optimize the sub-
system and do not consider marginal trade-offs. Therefore, such methods have no merit to serve
useful purpose.
4.14. SUMMARY
1. Read the statement of the problem. Confirm whether you have to maximize the objective
function or minimize the objective function.
2. Construct the transportation matrix.
3. Check whether the given problem is balanced or not.
4. If balanced proceed further. If not balanced, balance the problem by opening a dummy row
or a dummy column depending on the need. Let the unit cost of transportation of cells of
dummy row or column be zero.
Linear Programming: II Transportation Model 201
5. If the problem is maximization one convert that into a minimization problem by multiplying
the matrix by –1 or by subtracting all the elements of the matrix form the highest element in
the matrix.
6. Find the basic feasible solution. The characteristics of the basic feasible solution are it must
have (m + n – 1) allocations, where m is the number of rows and n is the number of columns.
7. The basic feasible solution may be obtained by (a) Northwest corner method, (b) Least Cost
method or Matrix minimum method, or (c) Vogel's approximation method or Opportunity
cost method.
8. If initial allocations are equal to (m + m – 1) proceed to next step. If it is not equal to (m + n
- 1) it is known as degenerate solution.
9. To solve degeneracy, add a small and negligible element ∈ to empty cells. Take care to see
that the ∈ loaded cell do not make closed loop with other loaded cells when lines are drawn
from epsilon loaded cells to other loaded cells by travelling vertically and horizontally by
taking turns at loaded cells.
10. Write allocations and calculate the total cost of transportation.
11. Give optimality test to the basic feasible solution. Optimality test can be given by (a) Stepping
stone method or (b) Modified distributing method or MODI method.
13. The characteristic of optimal solution is the opportunity costs of all empty cells are either
negatives or zeros.
14. Remember if any empty cell has zero as its opportunity cost, then we can write alternate
optimal solutions.
15. Write the allocations and calculate total transportation cost.
16. In case, the unit cost of transportation of any cell is zero or negative elements, take the same
into considerations for further calculations. Suppose nothing is given in the cell as the unit
cost of transportation, then presume that the route connecting the origin and the destination
through that cell is not existing and cancel that cell and do not consider it at all while solving
the problem, or else allocate very high cost of unit cost of transportation (infinity or any
number which is greater than all the elements in the matrix), so that that cell will not enter
into programme. (In maximization problem allocate a negative profit or return to the cell).
Problem 4.13. A company has three factories X, Y, and Z and four warehouses A, B, C, and D. It is
required to schedule factory production and shipments from factories to warehouses in such a manner
so as to minimize total cost of shipment and production. Unit variable manufacturing costs (UVMC)
and factory capacities and warehouse requirements are given below:
Solution: We have to optimize production and shipment cost. Hence the transportation matrix elements
are the total of manufacturing cost plus transportation cost. For example, the manufacturing cost of
factory X is Rs. 10. Hence the transportation and shipment cost will be equal to 10 + 0, 10 +1, 10 +1
and 10 +2 respectively for warehouses A, B, C and D respectively. As the total available is 174 units
and the total demand is 120 units we have to open a dummy column with requirement of 54 units. The
production cum transportation matrix is given below:
Production cum transportation cost per unit in Rs.
A B C D DC Avail Roc
X 10 11 11 11 0 75 10
Y 12 13 14 12 0 32 12
Z 16 15 15 18 0 67 15
54
Req 65 24 16 15 54 174
Coc 2 2 3 1 0
(3)
A B D Avail Roc
X 10 11 11 75 0
24
Y 12 13 12 32 1
Z 16 15 18 13 0
Req 65 24 15 104
Coc 2 2 0
(4)
A D Avail Roc
X 10 11 35 2
35
Y 12 12 32 0
Z 16 18 13 2
Req 65 15 80
Coc 2 0
(5)
A D Avail Roc
Y 12 12 32 0
15
Z 16 18 13 2
Req 30 15 45
Coc 4 6
(6)
A Avail Roc
Y 12 17 0
17
Z 16 13 2
13
Req 30 30
Coc 4
204 Operations Research
As there are m + n – 1 allocations and all the opportunity costs of empty cells are negative, the
solution is optimal.
The optimal allocations are:
QUESTIONS
6. How do you say that a transportation model has an alternate solution? In case it has an
alternate optimal solution, how do you arrive at alternate solution?
7. What is transshipment problem? In what way it differs from general transportation problem?
8. Explain the terms: (a) Opportunity cost, (b) Implied cost, (c) Row opportunity cost,
(d) Column opportunity cost.
9. The DREAM - DRINK Company has to work out a minimum cost transportation schedule
to distribute crates of drinks from three of its factories X, Y, and Z to its three warehouses A,
B, and C. The required particulars are given below. Find the least cost transportation schedule.
Transportation cost in Rs per crate.
10. The demand pattern for a product at for consumer centers, A, B, C and D are 5000 units,
7000 units, 4000 units and 2000 units respectively. The supply for these centers is from
three factories X, Y and Z. The capacities for the factories are 3000 units, 6000 units and
9000 units respectively. The unit transportation cost in rupees from a factory to consumer
center is given below in the matrix. Develop an optimal transportation schedule and find the
optimal cost.
From: To
A B C D
X 8 9 12 8
Y 3 4 3 2
Z 5 3 7 4
11. From three warehouses, A, B, and C orders for certain commodities are to be supplied to
demand points X, Y, and Z. Find the least cost transportation schedule with relevant information
given below:
X Y Z
A 5 10 2 100
B 3 7 5 25
C 6 8 4 75
Units demand: 105 30 90
206 Operations Research
12. From three warehouses A, B, and C orders for certain commodities are to be supplied to
demand points 1, 2, 3, 4 and 5 monthly. The relevant information is given below:
Warehouses Demand points (Transportation cost in Rs per unit. Availability in units.
1 2 3 4 5
A 4 1 2 6 9 100
B 6 4 3 5 7 120
C 5 2 6 4 8 120
Units demand: 40 50 70 90 90
During certain month a bridge on the road-connecting warehouse B to demand point 3 is closed
for traffic. Modify the problem suitably and find the least cost transportation schedule. (The demand
must be complied with).
13. A tin box company has four factories that supply to 5 warehouses. The variable cost of
manufacturing and shipment of one ton of product from each factory to each warehouse are
shown in the matrix given below, Factory capacities and warehouse requirements are shown
in the margin. After several iterations the solution obtained is also shown.
14. A company has a current shipping schedule, which is being questioned by the management
as to whether or not it is optimal. The firm has three factories and five warehouses. The
necessary data in terms of transportation costs in Rs. per unit from a factory to a destination
and factory capacities and warehouse requirements are as follows:
Solve for an optimal shipping schedule in terms of lowest possible shipping costs.
15. Solve the following transportation problem.
Destination
Source A B C D E Supply
W 20 19 14 21 16 40
X 15 20 13 19 16 60
Y 18 15 18 20 70
Z 0 0 0 0 0 50
Demand. 30 40 50 40 60
It is also possible to produce 25% higher than the capacity in each factory by working overtime
at 50% higher in direct costs.
(a) Build a transportation model so that the total demand is met with.
(b) Do the allocation of factory capacity by minimum cost allocation and check the solution
for optimality.
16. In a transportation problem the distribution given in the table below was suggested as an
optimal solution. The capacities and requirement are given. The number in bold are allocations.
The transportation costs given in Rs, per unit from a source to a destination.
(a) Test whether the given distribution is optimal?
(b) If not optimal obtain all basic optimal solution.
17. A department stores wishes to purchase 7,500 purses of which 2,500 are of style X, 2,500
are of style Y and 2,500 are of style Z. Four manufacturers A, B, C and D bid to supply not
more than the following quantities, all styles combined. A = 1,000, B = 3,000, C = 2, 100
and D = 1,900. The following table gives the cost per purse of each style of the bidders in
Rs. per purse.
MANUFACTURER.
Style A B C D
X 10 4 9 5
Y 6 7 8 7
Z 3 8 6 9
Linear Programming: II Transportation Model 209
(a) How should orders to be placed by the department store to minimize the total cost?
(b) If the store were to introduce a new style W, which manufacturer can supply it? How
many of W can he supply?
(c) Allocate the smallest element epsilon in such a cell, which will not form a closed loop
with other loaded cells.
(d) Allocate the smallest element epsilon in such a cell, which will form a closed loop with
other loaded cells. ( )
16. A problem where the produce of a factory is stored in warehouses and then they are transported
to various demand point as and when the demand arises is known as:
(a) Transshipment problem
(b) Warehouse problem
(c) Storing and transport problem
(d) None of the above ( )
17. Implied Cost in transportation problem sets (in the existing program):
(a) The lowest limit for the empty cell beyond which it is not advisable to include in the
programme,
(b) The highest limit for the empty cell beyond which it is not advisable to include in the
programme,
(c) The opportunity cost of the empty cell,
(d) None of the above. ( )
18. In transportation model, the opportunity cost is given by
(a) Implied cost + Actual cost of the cell
(b) Actual cost of the cell – Implied cost,
(c) Implied cost – Actual cost of the cell
(d) Implied cost × Actual cost of the cell ( )
19. If ui and vj are row and column numbers respectively, then the implied cost is given by:
(a) ui + vj (b) ui – vj
(c) ui × vj (d) ui /vj ( )
20. If a transportation problem has an alternate solution, then the other alternate solutions are
derived by:
(Given that the two matricides of alternate solutions are A and B, and d is any positive
fraction number)
(a) A + (1 – d) × B (b) A ( 1 – d) + B
(c) dA + dB (d) dA + (1 – d) × B ( )
ANSWERS
5.1. INTRODUCTION
In earlier discussion in chapter 3 and 4, we have dealt with two types of linear programming problems,
i.e. Resource allocation method and Transportation model. We have seen that though we can use
simplex method for solving transportation model, we go for transportation algorithm for simplicity.
We have also discussed that how a resource allocation model differ from transportation model and
similarities between them. Now we have another model comes under the class of linear programming
model, which looks alike with transportation model with an objective function of minimizing the time
or cost of manufacturing the products by allocating one job to one machine or one machine to one job
or one destination to one origin or one origin to one destination only. This type of problem is given the
name ASSIGNMENT MODEL. Basically assignment model is a minimization model. If we want to
maximize the objective function, then there are two methods. One is to subtract all the elements of the
matrix from the highest element in the matrix or to multiply the entire matrix by –1 and continue with
the procedure. For solving the assignment problem we use Assignment technique or Hungarian method
or Flood's technique. All are one and the same. Above, it is mentioned that one origin is to be assigned
to one destination. This feature implies the existence of two specific characteristics in linear programming
problems, which when present, give rise to an assignment problem. The first one being the pay of
matrix for a given problem is a square matrix and the second is the optimum solution (or any
solution with given constraints) for the problem is such that there can be one and only one assignment
in a given row or column of the given payoff matrix. The transportation model is a special case of
linear programming model (Resource allocation model) and assignment problem is a special case of
transportation model, therefore it is also a special case of linear programming model. Hence it must
have all the properties of linear programming model. That is it must have: (i) an objective function, (ii)
it must have structural constraints, (iii) It must have non-negativity constraint and (iv) The relationship
between variables and constraints must have linear relationship. In our future discussion, we will see
that the assignment problem has all the above properties.
(i) Assigning the jobs to machines when the problem has square matrix to minimize the time
required to complete the jobs. Here the number of rows i.e. jobs are equals to the number of
machines i.e. columns. The procedure of solving will be discussed in detail in this section.
(ii) The second type is maximization type of assignment problem. Here we have to assign
certain jobs to certain facilities to maximize the returns or maximise the effectiveness. This
is also discussed in problem number 5.2.
(iii) Assignment problem having non-square matrix. Here by adding a dummy row or dummy
columns as the case may be, we can convert a non-square matrix into a square matrix and
proceed further to solve the problem. This is done in problem number.5.9.
(iv) Assignment problem with restrictions. Here restrictions such as a job cannot be done on a
certain machine or a job cannot be allocated to a certain facility may be specified. In such
cases, we should neglect such cell or give a high penalty to that cell to avoid that cell to enter
into the programme.
(v) Traveling sales man problem (cyclic type). Here a salesman must tour certain cities starting
from his hometown and come back to his hometown after visiting all cities. This type of
problem can be solved by Assignment technique and is solved in problem 5.14.
Let us take that there are 4 jobs, W, X, Y and Z which are to be assigned to four machines, A, B,
C and D. Here all the jobs have got capacities to machine all the jobs. Say for example that the job W is
to drill a half and inch hole in a Wooden plank, Job X is to drill one inch hole in an Aluminum plate and
Job Y is to drill half an inch hole in a Steel plate and job Z is to drill half an inch hole in a Brass plate.
The machine A is a Pillar type of drilling machine, the machine B is Bench type of drilling machine,
Machine C is radial drilling machine and machine D is an automatic drilling machine. This gives an
understanding that all machines can do all the jobs or all jobs can be done on any machine. The cost or
time of doing the job on a particular machine will differ from that of another machine, because of
overhead expenses and machining and tooling charges. The objective is to minimize the time or cost of
manufacturing all the jobs by allocating one job to one machine. Because of this character, i.e. one to
one allocation, the assignment matrix is always a square matrix. If it is not a square matrix, then the
problem is unbalanced. Balance the problem, by opening a dummy row or dummy column with its
cost or time coefficients as zero. Once the matrix is square, we can use assignment algorithm or
Flood's technique or Hungarian method to solve the problem.
Jobs Machines (Time in hours) Availability
A B C D
W C 11 C 12 C 13 C 14 1
X C 21 C 22 C 23 C 24 1
Y C 31 C 32 C 33 C 34 1
Z C 41 C 42 C 43 C 44 1
Requirement: 1 1 1 1
Mathematical Model:
n n
Minimize Z = ∑ ∑C
i =1 j =1
ij × ij Objective Constraint.
214 Operations Research
∑
J =1
X ij = 1 (d ) and
j ∑X
i =1
ij = 1 (bi) Structural Constraints.
For i and j = 1 to n
(Each machine to one job only) (Each job to one machine only)
And
Xij = 0 for all values of j and i. Non-negativity constraint.
Similarities
1. Both are special types of linear programming problems.
2. Both have objective function, structural constraints, and non-negativity constraints. And
the relationship between variables and constraints are linear.
3. The coefficients of variables in the solution will be either 1 or zero in both cases.
4. Both are basically minimization problems. For converting them into maximization problem
same procedure is used.
Differences
Transportation Problem Assignment Problem.
1. The problem may have rectangular matrix 1.The matrix of the problem must be a square matrix.
or square matrix. 2.The rows and columns must have one to one
2.The rows and columns may have any allocation. Because of this property, the matrix must
number of allocations depending on the rim be a square matrix.
conditions. 3.The basic feasible solution is obtained by Hungarian
3.The basic feasible solution is obtained by method or Flood's technique or by Assignment
northwest corner method or matrix minimum algorithm.
method or VAM 4.Optimality test is given by drawing minimum
4.The optimality test is given by stepping number of horizontal and vertical lines to cover all
stone method or by MODI method. the zeros in the matrix.
5.The basic feasible solution must have m + 5.Every column and row must have at least one zero.
n – 1 allocations. And one machine is assigned to one job and vice versa.
6.The rim requirement may have any 6. The rim requirements are always 1 each for every
numbers (positive numbers). row and one each for every column.
7.In transportation problem, the problem 7.Here row represents jobs or machines and columns
deals with one commodity being moved from represents machines or jobs.
various origins to various destinations.
Linear Programming - III Assignment Model 215
Problem 5.1.
There are 3 jobs A, B, and C and three machines X, Y, and Z. All the jobs can be processed on all
machines. The time required for processing job on a machine is given below in the form of matrix.
Make allocation to minimize the total processing time.
Machines (time in hours)
Jobs X Y Z
A 11 16 21
B 20 13 17
C 13 15 12
C 13 15 12
Jobs X Y Z
A 11 16 21
B 20 13 17
C 13 15 12
Like this we have to write all allocations and calculate the cost and select the lowest one. If more
than one assignment has same lowest cost then the problem has alternate solutions.
Jobs X Y Z Available
A 1 E 1
B 1 ∈ 1
C 1 1
Req 1 1 1 3
As the basic feasible solution must have m + n – 1 allocations, we have to add 2 epsilons. Next we
have to apply optimality test by MODI to get the optimal answer.
This is a time consuming method. Hence it is better to go for assignment algorithm to get the
solution for an assignment problem.
Jobs X Y Z
A 11 16 21
B 20 13 17
C 13 15 12
Step 1. Deduct the smallest element in each row from the other elements of the row. The matrix
thus got is known as Row opportunity cost matrix (ROCM). The logic here is if we
assign the job to any machine having higher cost or time, then we have to bear the
penalty. If we subtract smallest element in the row or from all other element of the
row, there will be at least one cell having zero, i.e zero opportunity cost or zero
penalty. Hence that cell is more competent one for assignment.
Linear Programming - III Assignment Model 217
Step 2. Deduct the smallest element in each column from other elements of the column. The matrix
thus got is known as Column opportunity cost matrix (COCM). Here also by creating
a zero by subtracting smallest element from all other elements we can see the penalty
that one has to bear. Zero opportunity cell is more competent for assignment.
Step 3. Add COCM and ROCM to get the Total opportunity cost matrix (TOCM).
Step 4. (modified): Total opportunity cost matrix can be got by simplify doing row operation on
Column opportunity matrix or column operation on row opportunity cost matrix. This method
is simple one and saves time. (Doing row operation on column opportunity matrix means:
Deduct the smallest element in the row from all other elements in the row in column opportunity
matrix and vice versa).
The property of total opportunity cost matrix is that it will have at least one zero in
every row and column. All the cells, which have zero as the opportunity cost, are
eligible for assignment.
Step 5. Once we get the total opportunity cost matrix, cover all the zeros by MINIMUM NUMBER
OF HORIZONTAL AND VERTICAL LINES. (First cover row or column, which is having
maximum number of zeros and then next row or column having next highest number of
zeros and so on until all zeros are covered. Remember, only horizontal and vertical lines are
to be drawn.
Step 6. If the lines thus drawn are equal to the number of rows or columns (because of square
matrix), we can make assignment. If lines drawn are not equal to the number of rows or
columns go to step 7.
Step 7. To make assignment: Search for a single zero either row wise or column wise. If you start
row wise, proceed row by row in search of single zero. Once you find a single zero; assign
that cell by enclosing the element of the cell by a square. Once all the rows are over, then
start column wise and once you find single zero assign that cell and enclose the element of
the one cell in a square. Once the assignment is made, then all the zeros in the row and
column corresponding to the assigned cell should be cancelled. Continue this procedure until
all assignments are made. Some times we may not find single zero and find more than one
zero in a row or column. It indicates, that the problem has an alternate solution. We can
write alternate solutions. (The situation is known as a TIE in assignment problem).
Step 8. If the lines drawn are less than the number of rows or columns, then we cannot make
assignment. Hence the following procedure is to be followed:
The cells covered by the lines are known as Covered cells. The cells, which are not covered
by lines, are known as uncovered cells. The cells at the intersection of horizontal line and
vertical lines are known as Crossed cells.
(a) Identify the smallest element in the uncovered cells.
(i) Subtract this element from the elements of all other uncovered cells.
(ii) Add this element to the elements of the crossed cells.
(iii) Do not alter the elements of covered cells.
(b) Once again cover all the zeros by minimum number of horizontal and vertical lines.
(c) Once the lines drawn are equal to the number of rows or columns, assignment can be made
as said in step (6).
218 Operations Research
(d) If the lines are not equal to number of rows or columns, repeat the steps 7 (a) and 7 (b) until
we get the number of horizontal and vertical lines drawn are equal to the number of rows or
columns and make allocations as explained in step (6).
Note: For maximization same procedure is adopted, once we convert the maximization problem
into minimization problem by multiplying the matrix by (-1) or by subtracting all the elements of the
matrix from highest element in the matrix. Once we do this, the entries in the matrix gives us relative
costs, hence the problem becomes minimisaton problem. Once we get the optimal assignment, the
total value of the original pay off measure can be found by adding the individual original entries for
those cells to which assignment have been made.
Now let us take the problem given above and solve.
Solution
Jobs X Y Z
A 11 16 21
B 20 13 17
C 13 15 12
Step1: To find ROCM.
Machines (time in hours)
Jobs X Y Z
A 0 5 10
B 7 0 4
C 1 3 0
Jobs X Y Z
A 0 5 10
B 7 0 4
C 1 3 0
Because in each column, zero is the lowest element, the matrix remains unchanged, i.e. The
COCM itself TOCM.
Step 3. To cover all the zeros by minimum number of horizontal and vertical lines.
Machines (time in hours)
○
○
Jobs X Y Z
○
A 0 5 10
○
○ ○ ○ ○ ○ ○ ○ ○ ○ ○ ○ ○ ○ ○ ○ ○ ○ ○ ○ ○ ○
○
○ ○
B
○ ○ ○ ○ ○ ○
7
○ ○ ○ ○
0
○ ○ ○ ○ ○
4
○ ○ ○ ○
○
○
C 1 3 0
○
○
Linear Programming - III Assignment Model 219
Assignment is:
Machines (time in hours)
Jobs X Y Z
A 0 5 10
B 7 0 4
C 1 3 0
Problem 5.2.
A company has five jobs V, W, X, Y and Z and five machines A, B, C, D and E. The given matrix
shows the return in Rs. of assigning a job to a machine. Assign the jobs to machines so as to maximize
the total returns.
Machines.
Returns in Rs.
Jobs A B C D E
V 5 11 10 12 4
W 2 4 6 3 5
X 3 12 5 14 6
Y 6 14 4 11 7
Z 7 9 8 12 5
Solution
As the objective function is to maximize the returns, we have to convert the given problem into
minimization problem.
Method 1. Here highest element in the matrix is 14, hence subtract all the element form 14 and write the
relative costs. (Transformed matrix).
Machines
Returns in Rs.
Jobs A B C D E
V 9 3 4 2 10
W 12 10 8 11 9
X 11 2 9 0 8
Y 8 0 10 3 7
Z 7 5 6 2 9
220 Operations Research
ROCM:
Machines
Returns in Rs.
Jobs A B C D E
V 7 1 2 0 8
W 4 2 0 3 1
X 11 2 9 0 8
Y 8 0 10 3 7
Z 5 3 4 0 7
By doing column operation on ROCM, we get the total opportunity cost matrix.
TOCM:
Machines
Returns in Rs.
○
○
Jobs A B C D E
○
○
V 3 1 2 0 7
○
W 0 2 0 3 0
○
○ ○ ○ ○ ○ ○ ○ ○ ○ ○ ○ ○ ○ ○ ○ ○ ○ ○ ○ ○ ○ ○ ○
○
X 7 2 9 0 7
○
○
Y 4 0 10 3 6
○
○ ○ ○ ○ ○ ○ ○ ○ ○ ○ ○ ○ ○ ○ ○ ○ ○ ○ ○ ○ ○ ○ ○ ○
○
Z 1 3 4 0 6
○
○
○
Only three lines are there. So we have to go to step 7. The lowest element in uncovered cell is 1,
hence subtract 1 from all uncovered cells and add this element to crossed cells and write the matrix.
The resultant matrix is:
Machines
Return in Rs.
○
○
Jobs A B C D E
○
○
V 2 0 1 0 6
○
○ ○ ○ ○ ○ ○ ○ ○ ○ ○ ○ ○ ○ ○ ○ ○ ○ ○ ○ ○ ○ ○ ○ ○
W 0 3 0 4 0
○
○
X 6 1 8 0 6
○
○
Y 4 0 10 4 6
○
○
○ ○ ○ ○ ○ ○ ○ ○ ○ ○ ○ ○ ○ ○ ○ ○ ○ ○ ○ ○ ○ ○ ○ ○
Z 0 2 3 0 5
○
○
○
Only foor lines are there, hence repeat the step 7 until we get 5 lines.
Linear Programming - III Assignment Model 221
Machines
Return in Rs.
○
○
Jobs A B C D E
○
○
V 1 0 0 0 5
○
○ ○ ○ ○ ○ ○ ○ ○ ○ ○ ○ ○ ○ ○ ○ ○ ○ ○ ○ ○ ○ ○ ○ ○
W 0 3 0 5 0
○
○
X 5 1 7 0 5
○
○
Y 3 0 9 4 5
○
○
○ ○ ○ ○ ○ ○ ○ ○ ○ ○ ○ ○ ○ ○ ○ ○ ○ ○ ○ ○ ○ ○ ○ ○
Z 0 3 3 1 5
○
○
○
All zeros are covered by 5 lines, Hence assignment can be made. Start row wise or column wise
and go on making assignment, until all assignments are over.
Machines
Return in Rs.
Jobs A B C D E
V 2 1 0 x0 5
W 1 4 0x 5 0
X 6 2 7 0 5
Y 3 0 8 3 4
Z 0 3 2 0x 4
Problem 5.3.
Five jobs are to be assigned to 5 machines to minimize the total time required to process the jobs
on machines. The times in hours for processing each job on each machine are given in the matrix
below. By using assignment algorithm make the assignment for minimizing the time of processing.
Machines (time in hours)
Jobs V W X Y Z
A 2 4 3 5 4
B 7 4 6 8 4
C 2 9 8 10 4
D 8 6 12 7 4
E 2 8 5 8 8
222 Operations Research
Solution
Machines (time in hours)
Jobs V W X Y Z
A 2 4 3 5 4
B 7 4 6 8 4
C 2 9 8 10 4
D 8 6 12 7 4
E 2 8 5 8 8
COCM
Machines (time in hours)
○
○
Jobs V W X Y Z
○
○
○
○
○ ○ ○ ○ ○ ○ ○ ○ ○ ○ ○ ○ ○ ○ ○ ○ ○ ○ ○ ○ ○ ○ ○
A 0 0 0 0 0
○
○
○
○
B 5 0 3 3 0
○
○
○ ○ ○ ○ ○ ○ ○ ○ ○ ○ ○ ○ ○ ○ ○ ○ ○ ○ ○ ○ ○ ○ ○
○
○
C 0 5 5 5 0
○
D 6 2 9 2 ○
0
○
○
○
E 0 4 2 3 4
○
○
○
○
○
As the COCM has at least one zero in every column and row, this itself can be considered as
TOCM, because as the zero is the lowest number in each column, the matrix remains unchanged. If we
cover all the zeros by drawing horizontal and vertical lines, we get only four lines. Applying step 7 we
get the following matrix.
Machines (time in hours)
○
○
○
○
Jobs V W X Y Z
○
○
A 2 0 0 0 2
○
○
○ ○ ○ ○ ○ ○ ○ ○ ○ ○ ○ ○ ○ ○ ○ ○ ○ ○ ○ ○ ○ ○ ○
○
○
B 7 0 3 3 2
○
○
○
○
○ ○ ○ ○ ○ ○ ○ ○ ○ ○ ○ ○ ○ ○ ○ ○ ○ ○ ○ ○ ○ ○ ○
C 0 3 3 3 0
○
○
○
○
○ ○ ○ ○ D ○ ○ ○ ○ 6
○ ○ ○ 0
○ ○ ○ ○ 7 ○ ○ ○ ○ 0 ○ ○ ○ 0 ○ ○ ○ ○
○
○
○
E 0 2 0 1 4
○
○
○
As there are five lines that cover all zeros, we can make assignment.
Machines (time in hours)
Jobs V W X Y Z
A 2 0 0 0 2
B 7 0 3 3 2
C 0 3 3 3 0
D 6 0 7 0 0
E 0 2 0 1 4
Linear Programming - III Assignment Model 223
Alternate solution:
Machines (time in hours)
Jobs V W X Y Z
A 2 0 0 0 2
B 7 0 3 3 2
C 0 3 3 3 0
D 6 0 7 0 0
E 0 2 0 1 4
Problem 5.4.
A manager has 4 jobs on hand to be assigned to 3 of his clerical staff. Clerical staff differs in
efficiency. The efficiency is a measure of time taken by them to do various jobs. The manager wants
to assign the duty to his staff, so that the total time taken by the staff should be minimum. The matrix
given below shows the time taken by each person to do a particular job. Help the manager in assigning
the jobs to the personnel.
Jobs. Men (time taken to do job in hours).
X Y Z
A 10 27 16
B 14 28 7
C 36 21 16
D 19 31 21
Solution
The given matrix is unbalanced. To balance the matrix, open a dummy column with time coefficients
as zero.
(DC = Dummy column).
Men (Time taken in hours)
X Y Z DC
A 10 27 16 0
B 14 28 7 0
C 36 21 16 0
D 19 31 21 0
224 Operations Research
As every row has a zero, we can consider it as ROCM and by doing column operation, we can
write TOCM. Now apply step 7.
Men (Time taken in hours).
○
○
Jobs X Y Z DC
○
○ ○ ○ ○ ○ ○ ○ ○ ○ ○ ○ ○ ○ ○ ○ ○ ○ ○ ○ ○ ○ ○ ○
A 0 6 9 0
○
○
B 4 7 0 0
○
○ ○ ○ ○ ○ ○ ○ ○ ○ ○ ○ ○ ○ ○ ○ ○ ○ ○ ○ ○ ○ ○ ○
○
○ ○ ○ C ○ ○ ○ 26
○ ○ ○ ○ ○ 0○ ○ ○ ○ 9○ ○ ○ ○ 0○ ○ ○ ○
○
○
D 9 10 14 0
○
○
Men (Time taken in hours).
Jobs X Y Z DC
A 0 6 9 0
B 4 7 0 0
C 26 0 9 0
D 9 10 14 0
Problem 5.5.
A company has four market segments open and four salesmen are to be assigned one to each
segment to maximize the expected total sales. The salesmen differ in their ability and the segments also
differ in their sales potential. The details regarding the expected sales in each segment by a typical
salesman under most favourable condition are given below.
Segment A = Rs. 60,000, Segment B = Rs. 50,000, Segment C = Rs. 40,000 and Segment D =
Rs. 30,000. It is estimated that working under same condition, the ability of salesmen in terms of
proportional yearly sales would be as below:
Salesman W = 7, Salesman X = 5, Salesman Y = 5 and Salesman Z = 4.
Assign segments to salesmen for maximizing the total expected sales.
Solution
To simplify the calculations, let us consider sales of Rs.10, 000/- as one unit of sale, then salesman
W ’s annual sales in four segments are:
His proportionate sale is seven out of 21 (7 + 5 + 5 + 4 = 21). In case the annual sales is 6 units
(Rs.60, 000), then his proportional sales would be (7 / 21) × 6 = 42 / 21 similarly his sales in all
the segments would be (7/21) × 6, (7/21) × 5, (7/21) × 5, and (7/21) × 4 i.e. 42/21, 35/21, 35/21 and
28/21. Like wise we can calculate the proportional sales of all salesmen and write the matrix showing
the sales of each salesman in different market segments. The matrix is given below:
Linear Programming - III Assignment Model 225
Market segments.
Sales (x1000) 6 5 4 3
Salesproportion Salesmen W X Y Z
7 W 42/21 3 5/21 28/21 21/21
5 X 30/21 25/21 20/21 15/21
5 Y 30/21 25/21 20/21 15/21
4 Z 24/21 20/21 16/21 12/21
Multiply the matrix by21 to avoid the denominator. As the problem is maximization one, convert
the problem into minimization problem by multiplying by (-1) (Second method). The resultant matrix
is:
Market segments.
SalesMen A B C D
W –42 –35 –28 –21
X –30 –25 –20 –15
Y –30 –25 –20 –15
Z –24 –20 –16 –12
ROCM:
Market segments.
SalesMen A B C D
W 0 7 14 21
X 0 5 10 15
Y 0 5 10 15
Z 0 4 8 12
TOCM:
Market segments.
○
SalesMen A B C D
○
○
W 0 3 6 9
○
○
X 0 1 2 3
○
○
Y 0 1 2 3
○
○ ○ ○ ○ ○ ○ ○ ○ ○ ○ ○ ○ ○ ○ ○ ○ ○ ○ ○ ○ ○ ○ ○
Z 0 0 0 0
○
○
○
226 Operations Research
TOCM:
Market segments.
○
SalesMen A B C D
○
○
W 0 2 5 8
○
○
X 0 0 1 2
○
○
Y 0 0 1 2
○
○ ○ ○ ○ ○ ○ ○ ○ ○ ○ ○ ○ ○ ○ ○ ○ ○ ○ ○ ○ ○ ○ ○ ○ ○
○
Z 1 0 0 0
○
○
TOCM:
○
Market segments.○
○
○
○
SalesMen A B C D
○
○
○
○
W 0 2 4 7
○
○
○
○
X 0 0 0 1
○
○
○
Y 0 0 0 1
○
○
○
○ ○ ○ ○ ○ ○ ○ ○ ○ ○ ○ ○ ○ ○ ○ ○ ○ ○ ○ ○ ○ ○ ○ ○ ○
○
Z 2 1 0 0
○
○
○
○
Market segments.
SalesMen A B C D
W 0 2 4 7
X 0x 0 0x 1
Y 0x 0x 0 1
Z 2 1 0x 0
(Alternate Solution)
Market segments.
SalesMen A B C D
W 0 2 4 7
X 0x 0x 0 1
Y 0x 0 0x 1
Z 2 1 0x 0
Problem 5.6.
The city post office has five major counters namely, Registration (R), Savings (S), Money –
Order (M), Postal stationary (P) and Insurance / license (I). The postmaster has to assign five counters
Linear Programming - III Assignment Model 227
to five clerks A, B, C, D and E one for each counter. Considering the experience and ability of these
clerks he rates their suitability on a certain 10 - point scale of effectiveness of performance for
accomplishing different counter duties, as listed below. Assign the counters to the clerks for maximum
effective performance.
Clerks (effective performance)
Counters A B C D E
R 6 6 4 6 7
S 5 4 3 6 8
M 7 6 3 5 5
P 7 5 6 8 8
I 4 3 6 7 6
Convert the problem into minimization problem. (We can deduct all other elements form highest
element).
Note : As every row has a zero, we can consider it as Row Opportunity Cost Matrix.
ROCM
Clerks (effective performance)
Counters A B C D E
R 1 1 3 1 0
S 3 4 5 2 0
M 0 1 4 2 2
P 1 3 2 0 0
I 3 4 1 0 1
TOCM:
Clerks (effective performance)
○
Counters A B C D E
○
○
○
R 1 0 2 1 0
○
○
○
S 3 3 4 2 0
○
○
○
M 0 0 3 2 0
○
○ ○ ○ ○ ○ ○ ○ ○ ○ ○ ○ ○ ○ ○ ○ ○ ○ ○ ○ ○ ○ ○ ○ ○ ○ ○ ○ ○
○
P 1 2 1 0 0
○
○ ○ ○
I ○ ○ ○ ○ ○ ○ ○
3
○ ○ ○
3
○ ○ ○ ○
0
○ ○ ○
0
○ ○ ○
1
○ ○ ○ ○ ○
○
○
○
○
○
○
228 Operations Research
Counters A B C D E
R 1 0 2 1 0x
S 3 3 4 2 0
M 0 0x 3 2 2
P 1 2 1 0 0x
I 3 3 0 0x 1
Problem 5.7.
There are 5 jobs namely, A, B, C, D, and E. These are to be assigned to 5 machines P, Q, R, S and
T to minimize the cost of production. The cost matrix is given below. Assign the jobs to machine on
one to one basis.
Jobs (Cost in Rs.)
Machines A B C D E
P 8 7 4 11 6
Q 10 5 5 13 7
R 6 9 8 7 12
S 6 7 2 3 2
T 7 8 8 10 5
ROCM:
Jobs (Cost in Rs.)
Machines A B C D E
P 4 3 0 7 2
Q 5 0 0 8 2
R 0 3 2 1 6
S 4 5 0 1 0
T 2 3 3 5 0
Linear Programming - III Assignment Model 229
TOCM:
Jobs (Cost in Rs.)
○
○
○
Machines A B C D E
○
○
○
P 4 3 0 6 2
○
○
○ ○ ○ ○ ○ ○ ○ ○ ○ ○ ○ ○ ○ ○ ○ ○ ○ ○ ○ ○ ○ ○ ○ ○ ○ ○ ○ ○ ○
Q 5 0 0 7 2
○
○
○
R 0 3 2 0 6
○
○ ○ ○ ○ ○ ○ ○ ○ ○ ○ ○ ○ ○ ○ ○ ○ ○ ○ ○ ○ ○ ○ ○ ○ ○ ○ ○ ○ ○ ○ ○
○
S 4 5 0 0 0
○
○ ○ ○ ○ ○ ○ ○ ○ ○ ○ ○ ○ ○ ○ ○ ○ ○ ○ ○ ○ ○ ○ ○ ○ ○ ○ ○ ○ ○ ○ ○
○
○
○
T 2 3 3 4 0
○
○
○
○
There are five lines and hence we can make assignment.
○
Jobs (Cost in Rs.)
Machines A B C D E
P 4 3 0 6 2
Q 5 0 0 7 2
R 0 3 2 0 6
S 4 5 0 0 0
T 2 3 3 4 0
Problem 5.8.
Four different jobs are to be done on four machines, one job on each machine, as set up costs and
times are too high to permit a job being worked on more than one machine. The matrix given below
gives the times of producing jobs on different machines. Assign the jobs to machine so that total time
of production is minimized.
Machines (time in hours)
Jobs A B C D
P 10 14 22 12
Q 16 10 18 12
R 8 14 20 14
S 20 8 16 6
230 Operations Research
Solution
ROCM:
Machines (time in hours)
Jobs A B C D
P 0 4 12 2
Q 6 0 8 2
R 0 6 12 6
S 14 2 10 0
TOCM:
Machines (time in hours)
○
Jobs A B C D
○
○
P 0 4 4 2
○
○
Q 6 0 0 2
○
○ ○ ○ ○ ○ ○ ○ ○ ○ ○ ○ ○ ○ ○ ○ ○ ○ ○ ○ ○
○
R 0 6 4 6
○
○
S 14 2 2 0
○
○ ○ ○ ○ ○ ○ ○ ○ ○ ○ ○ ○ ○ ○ ○ ○ ○ ○ ○ ○
TOCM:
○
Jobs A B C D
○
○
P 0 4 4 2
○
○
Q 6 0 0 2
○
○ ○ ○ ○ ○ ○ ○ ○ ○ ○ ○ ○ ○ ○ ○ ○ ○ ○ ○ ○
○
R 0 6 4 6
○
○
S 14 2 6 0
○
TOCM:
○
Jobs A B C D
○
○
P 0 2 2 0
○
○
○ ○ ○ ○ ○ ○ ○ ○ ○ ○ ○ ○ ○ ○ ○ ○ ○ ○ ○ ○ ○
Q 8 0 0 2
○
○
R 0 4 2 4
○
○
S 16 2 4 0
○
○
Linear Programming - III Assignment Model 231
TOCM:
Machines (time in hours)
Jobs A B C D
○ ○ ○ ○ ○ ○ ○ ○ ○ ○ ○ ○ ○ ○ ○ ○ ○ ○ ○ ○ ○
P 0 0 0 0
○ ○
Q
○ ○ ○ ○
10
○ ○ ○ ○
0
○ ○ ○
0
○ ○ ○ ○
4 ○ ○ ○
○ ○
R
○ ○ ○ ○
0
○ ○ ○ ○
2
○ ○ ○
0
○ ○ ○
0
○ ○ ○ ○
○ ○ ○ ○ ○ ○ ○ ○ ○ ○ ○ ○ ○ ○ ○ ○ ○ ○ ○ ○ ○ ○ ○
S 16 0 0 0
Four lines are there hence we can make assignment. As there is a tie, we have more than one
solution.
Solution I.
TOCM:
Machines (time in hours)
Jobs A B C D
P 0 0 0 0
Q 10 0 0 4
R 0 2 0 0
S 14 0 4 0
Assignment: P to A, Q to B, R to C and S to D.
Time: 10 + 10 + 20 + 06 = 46 hours.
Solution II.
TOCM:
Machines (time in hours)
Jobs A B C D
P 0 0 0 0
Q 10 0 0 4
R 0 2 0 0
S 14 0 4 0
Assignment: P to B, Q to C, R to A and S to D.
Time: 14 + 18 + 8 + 6 = 46 hours.
We can write many alternate solutions.
232 Operations Research
Problem 5.9.
On a given day District head quarter has the information that one ambulance van is stationed at each of
the five locations A, B, C, D and E. The district quarter is to be issued for the ambulance van to reach
6 locations namely, P, Q, R, S, T and U, one each. The distances in Km. between present locations of
ambulance vans and destinations are given in the matrix below. Decide the assignment of vans for
minimum total distance, and also state which destination should not expect ambulance van to arrive.
To (distance in Km.)
From P Q R S T U
A 18 21 31 17 26 29
B 16 20 18 16 21 31
C 30 25 27 26 18 19
D 25 33 45 16 32 20
E 36 30 18 15 31 30
Solution
As the given matrix is not square matrix, balance the same by opening one dummy row (DR),
with zero as the elements of the cells.
To (distance in Km.)
From P Q R S T U
A 18 21 31 17 26 29
B 16 20 18 16 21 31
C 30 25 27 26 18 19
D 25 33 45 16 32 20
E 36 30 18 15 31 30
DR 0 0 0 0 0 0
As every column has got one zero, we can take it as COCM. Now doing row operation on
COCM, we get TOCM.
TOCM
○
○
P Q R S T U
○
A 1 4 14 0 9 12
○
○
○ ○ ○ ○
B ○ ○ ○ ○
0 ○ ○ ○ ○
4 ○ ○ ○
2 ○ ○ ○ ○
0 ○ ○ ○
5 ○ ○ ○
15 ○ ○ ○ ○ ○
○
○
C 12 7 9 8 0 1
○
○ ○ ○ ○ ○ ○ ○ ○ ○ ○ ○ ○ ○ ○ ○ ○ ○ ○ ○ ○ ○ ○ ○ ○ ○ ○ ○ ○ ○ ○
○
D 9 17 29 0 16 4
○
E 21 15 3 0 16 15
○
○
DR 0 0 0 0 0 0
○
○ ○ ○ ○ ○ ○ ○ ○ ○ ○ ○ ○ ○ ○ ○ ○ ○ ○ ○ ○ ○ ○ ○ ○ ○ ○ ○ ○ ○ ○ ○ ○
○
○
Linear Programming - III Assignment Model 233
○
○
○
P Q R S T U
○
○
○
A 0 3 13 0 8 11
○
○
B 0 4 2 0 5 15
○
○
○
○
○ ○ ○ ○ ○ ○ ○ ○ ○ ○ ○ ○ ○ ○ ○ ○ ○ ○ ○ ○ ○ ○ ○ ○ ○ ○ ○
C 12 7 9 9 0 1
○
○
○
○
D 8 16 28 0 15 3
○
○
○
○
E 20 14 2 0 15 14
○
○
○ ○ ○ ○ ○ ○ ○ ○ ○ ○ ○ ○ ○ ○ ○ ○ ○ ○ ○ ○ ○ ○ ○ ○ ○ ○ ○
○
○
DR 0 ○
0 0 1 0 0
○
○
○
As there are four lines, we cannot make an assignment.
To (Distance in Km.)
○
○
○
○
○
P Q R S T U
○
○
○
A 0 1 11 0 6 9
○
○
○
○
B 0 2 0 1 3 13
○
○
○
○
○
C 14 7 9 11 0 1
○
○ ○ ○ ○ ○ ○ ○ ○ ○ ○ ○ ○ ○ ○ ○ ○ ○ ○ ○ ○ ○ ○ ○ ○ ○ ○
○
○
○
○
D 8 14 26 0 13 1
○
○
○
○
○
E 20 12 0 0 13 12
○
○
○
○
○
○ ○ DR ○ ○ ○ ○ 2
○ ○ ○ ○ 0 ○ ○ ○ 0 ○ ○ ○ ○ 3 ○ ○ ○ 0 ○ ○ ○ ○ 0 ○ ○ ○
○
○
○
○
○
P Q R S T U
○
○
○ ○ ○
A ○ ○ ○ ○ ○
0 ○ ○ ○
0 ○ ○ ○
11 ○ ○ ○ ○
0 ○ ○ ○
5 ○ ○ ○ ○
8 ○ ○ ○ ○
○
○ ○ ○ ○ ○ ○ ○ ○ ○ ○ ○ ○ ○ ○ ○ ○ ○ ○ ○ ○ ○ ○ ○ ○ ○ ○ ○ ○
B 0 1 0 1 2 12
○
○
C 15 7 10 12 0 1
○
○
○ ○ ○
D ○ ○ ○ ○
8 ○ ○ ○
13 ○ ○ ○
26
○ ○ ○ ○
0
○ ○ ○ ○
12 ○ ○ ○
0
○ ○ ○ ○
○
○
○ ○ ○ ○ ○ ○ ○ ○ ○ ○ ○ ○ ○ ○ ○ ○ ○ ○ ○ ○ ○ ○ ○ ○ ○ ○ ○ ○
E 20 11 0 0 12 11
○
○
○ ○ DR ○ ○ ○ ○ 3 ○ ○ ○ 0
○ ○ ○ ○ 1 ○ ○ ○ 4 ○ ○ ○ 0
○ ○ ○ ○ 0 ○ ○ ○
○
○
As there are 6 lines, we can make assignment. As there is a tie, we have alternate solutions.
To (Distance in Km.)
From P Q R S T U
A 0 0 11 0 5 8
B 0 1 0 1 2 12
C 15 7 10 12 0 1
D 8 13 26 0 12 0
E 20 11 0 0 12 11
DR 4 0 1 4 0 0
234 Operations Research
Brain tonic:
a) In case the cost of dispatching an ambulance is 3 times the distance, determine the assignment
of ambulances to destinations.
(b) In case the operating cost of a van is proportional to the square of the distance decide the
assignment.
(Note: a) By multiplying the entire matrix by 3 we get the cost matrix. This does not have any effect
on the final solution. Hence the same solution will hold good.
(b) We have to write the elements by squaring the elements of the original matrix and make fresh
assignment.)
Problem 5.10.
A job order company has to work out the assignment of 5 different jobs on five different machines.
The cost of machining per unit of job and set up cost of the job on a machine are as given in the matrix
A and B given below. The jobs are to be made in bathe sizes show against them. Set up cost is
independent of previous set up.
Matrix A. (Operating cost in Rs)
Jobs (machining cost in Rs)
Machines. A B C D E
P 0.80 1.10 0.70 1.60 6.20
Q 1.20 0.90 1 .20 0.80 5.40
R 2.10 2.00 1.00 2.20 4.90
S --- 1.60 2.00 1.90 3.60
T 3.20 2.00 2.00 2.00 2.60
Batch size in units. 100 100 150 100 50
Machines A B C D E
P 60 70 70 30 40
Q 40 50 50 20 80
R 30 40 40 40 100
S --- 90 60 50 60
T 80 100 80 60 60
Linear Programming - III Assignment Model 235
Solution
Multiply the Matrix A by 100 and add it to the matrix B we get the matrix given below. For the element
SA as nothing is given, we can eliminate it for further consideration or assign a very high cost for the
element so as to avoid it from further calculations.
Jobs (combined setup and processing cost in Rs)
Machines A B C D E
P 140 180 175 190 350
Q 160 140 230 100 350
R 240 240 190 260 255
S 1000 250 360 240 240
T 400 300 380 260 190
ROCM:
Machines A B C D E
P 0 40 35 50 210
Q 60 40 130 0 250
R 50 50 0 70 65
S 760 10 120 0 0
T 210 110 190 70 0
TOCM:
Machines A B C D E
P 0 30 35 50 210
Q 60 30 130 0 250
R 50 40 0 70 65
S 760 0 120 0 0
T 210 100 190 70 0
Assignment: P to A, Q to D, R to C, S to B and T to E.
Total cost = 140 + 100 + 190 + 250 + 190 = Rs. 870/-
Problem 5.11.
There are five major projects namely, Fertiliser plants, Nuclear poser plants, Electronic park, Aircraft
complex and Heavy machine tools. These five plants are to be assigned to six regions namely A, B, C,
D, E and F, insisting on allocation of as many number of projects as possible in their region. The state
department has evaluated the effectiveness of projects in different regions for (a) Employment potential,
(b) Resource utilization potential, (c) Economic profitability and (d) Environmental degradation index
as given below in
236 Operations Research
Tableau I. (The ranking is on a 20 point scale). Assign one project to one region depending on the
maximum total effectiveness. (Plants are given serial numbers 1 to 5)
Tableau I.
Local Employment Resource Allocation Economic Profitability Environmental
Potential. Potential. Index. Degradation index.
Reg. 1 2 3 4 5 1 2 3 4 5 1 2 3 4 5 1 2 3 4 5
long.
A 16 10 8 12 11 7 6 4 5 3 11 13 14 15 10 15 14 5 3 2
B 18 15 12 10 7 11 4 3 2 1 10 15 17 11 16 13 14 5 3 2
C 12 16 12 5 8 16 5 4 3 2 13 14 16 12 11 12 11 5 4 2
D 14 10 13 6 8 15 3 2 4 1 7 10 5 11 8 12 11 5 4 2
E 15 17 11 18 11 8 3 4 2 4 10 12 7 11 16 9 6 5 4 3
F 12 18 11 15 1 4 17 5 2 1 3 5 10 12 13 12 6 3 5 5 2
Solution
In this problem, for maximization of total effectiveness, the first three i.e. Employment potential,
Resource utilization potential and economic profitability index are to be added and the environmental
degradation is to be subtracted from the sum to get the total effectiveness. Once we get the effectiveness
matrix, then the projects are to be assigned to the regions for maximization of total effectiveness.
The total effectiveness matrix: (Note: The matrix is of the order 5 × 6, hence it is to be balanced
by opening a dummy column - DC). The first element of the matrix can be worked out as: 16 + 7 + 11
– 15 = 19. Other elements can be worked out similarly.
Total effectiveness matrix:
Plants.
Regions 1 2 3 4 5 DC
A 19 15 21 29 22 0
B 26 20 27 20 22 0
C 29 24 27 16 19 0
D 24 12 15 17 15 0
E 24 26 17 27 28 0
F 28 30 20 24 27 0
As there is a dummy column the same matrix may be considered as ROCM. By deducting all the
elements of a column from the highest element of the column, we get the Total Opportunity Cost
Matrix.
Linear Programming - III Assignment Model 237
TOCM:
Plants.
Regions 1 2 3 4 5 DC
A 10 15 6 0 6 0x
B 3 10 0 9 6 0x
C 0 6 0x 13 11 0x
D 5 18 12 12 13 0
E 5 4 10 2 0 0x
F 2 0 7 5 1 0x
Solution
Now let us consider the layover times separately for crew based at Mumbai and crew based at
Bangalore.
Let us consider one flight and discuss how to calculate layover time. For example, flight No. 101
leaves Mumbai at 6.30 a.m and reaches Bangalore at 7.45 a.m. Unless the crew takes one our rest, they
cannot fly the airplane. So if the crew cannot leave Bangalore until 8.45 a.m. So there is no chance for
the crew to go for flight No. 102. But they can go as flight Nos. 103, 106, 108 and 110. As we have
to minimize the flyover time, we can take the nearest flight i.e. 103. The flight 103 leaves Bangalore at
11.00 a.m. By 11.00 a.m the crew might have spent time at Bangalore from 7.45 a.m to 11.00 a.m.
That is it has spent 3 hours and 15 minutes. If we convert 3 hours and 15 minutes in terms of quarter
hours, it will become 13-quarter hours. Similarly the flight 102 which arrives at Mumbai at 8.00 a.m.
wants to leave as flight 101 at 6.30 a.m. it has to leave next day morning. Hence the layover time will
be 22 hours and 30 minutes. Like wise, we can workout layover time for all flights and we can write
two matrices, one for crew at Mumbai and other for crew at Bangalore.
Tableau I. Lay over time for Mumbai based crew:
The matrices can be multiplied by four to convert decimals into whole numbers for convenience
of calculations.
Linear Programming - III Assignment Model 239
Now let us select the minimum elements from both the matrices and write another matrix with
these elements. As our objective is to minimize the total layover time, we are selecting the lowest
element between the two matrices. Also, let us mark a * for the entries of the matrix showing layover
time of the crew at Bangalore.
Matrix showing the lowest layover time
(The elements marked with * are from Bangalore matrix)
Flight No. 102 104 106 108 110
101 90* 13 29 40 35*
103 4* 83* 14 29 40
105 20* 83 83* 14 21
107 32* 15* 86 83* 12
109 38 31* 15* 81 89*
ROCM: As every column has got a zero, this may be considered as TOCM and assignment can be
made. Note that all zeros in the matrix are in independent position we can make assignment.
Flight No. 102 104 106 108 110
101 77* 0 16 27 22*
103 0* 79* 10 25 36
105 6* 69 69* 0 7
107 20* 3* 74 71* 0
109 23 16* 0* 65 74*
240 Operations Research
Problem 5.13.
An airline that operates seven days a week has the timetable shown below. Crews must have a
minimum layover time 5 hours between flights. Obtain the pairing of flights that minimises layover
time away from home. For any given pairing, the crew will be based at the city that results in the
smaller layover. For each pair also mention the town where crew should be based.
Chennai - Bangalore Bangalore - Chennai.
FlightNo. Departure Arrival Flight No. Departure Arrival
101 7.00 a.m 8.00 a.m 201 8.00 a.m 9.00 a.m
102 8.00 a.m 9.00 a.m 202 9.00 a.m 10.00 a.m
103 1.00 p.m 2.00 p.m 203 12.00 noon 1.00 p.m.
104 6.00 p.m. 7.00 p.m 204 8.00 p.m 9.00 p.m
Let us write two matrices one for layover time of Chennai based crew and other for Bangalore
based crew.
As explained in the example 5.11 the departure of the crew once it reaches the destination, should
be found after taking the minimum layover time given, i.e. 5 hours. After words, minimum elements
from both the matrices are to be selected to get the matrix showing minimum layover times. Finally, we
have to make assignment for minimum layover time.
Layover time for Chennai based crew in hours.
Tableau I.
FlightNo. 201 202 203 203
101 24 25 28 12
102 23 24 27 11
103 20 19 22 6
104 13 14 17 25
Linear Programming - III Assignment Model 241
Minimum of the two matrices layover time. The Bangalore based times are marked with a (*).
Tableau I.
FlightNo. 201 202 203 203
101 22* 21* 18* 10*
102 23** 22* 19* 11**
103 20 19 22 6
104 9* 8* 5* 21*
The elements with two stars (**) appear in both the matrices.
ROCM
Tableau I.
FlightNo. 201 202 203 203
101 12 11 8 0
102 12 11 8 0
103 14 13 16 0
104 4 3 0 16
TOCM:
○
○
101 8 8 8 0
○
○
102 8 8 8 0
○
○
103 6 10 16 0
○
○ ○ ○ ○ ○ ○ ○ ○ ○ ○ ○ ○ ○ ○ ○ ○ ○ ○ ○ ○ ○ ○ ○ ○ ○ ○
○
104 0 0 0 16
○
○
○
101 2 2 2 0
○
○
102 2 2 2 0
○
○
○ ○
103
○ ○ ○ ○ ○ ○ ○ ○
0 ○ ○ ○ ○
4 ○ ○ ○ ○
10 ○ ○ ○ ○
0
○ ○ ○ ○ ○
○
○
○ ○
104 ○ ○ ○ ○ ○ ○ ○
0 ○ ○ ○ ○
0
○ ○ ○ ○ ○
0 ○ ○ ○ ○
22 ○ ○ ○ ○ ○
○
○
○
242 Operations Research
○
○
FlightNo. 201 202 203 203
○
○ ○ ○ ○ ○ ○ ○ ○ ○ ○ ○ ○ ○ ○ ○ ○ ○ ○ ○ ○ ○ ○ ○ ○ ○ ○
101 0 0 0 0
○
○
102 0 0 0 0
○
○
○ ○ 103
○ ○ ○ ○ ○ ○ ○ ○ 0 ○ ○ ○ ○ 4 ○ ○ ○ ○ 10 ○ ○ ○ ○ 2
○ ○ ○ ○ ○
○
○ ○ ○ ○ ○ ○ ○ ○ ○ ○ ○ ○ ○ ○ ○ ○ ○ ○ ○ ○ ○ ○ ○ ○ ○ ○ ○
○
104 0 0 0 24
○
○
FlightNo. 201 202 203 204
101 0 0 0 0*
102 0 0* 0 0
103 0 4 10 2
104 0 0 0* 24
Assignment:
Flight No. Leaves as Based at
101 204 Bangalore
102 202 Bangalore
103 201 Chennai
104 203 Bangalore.
n n
∑X
J =1
ij = 1 for i = 1,2,….n (Depart from a city once only)
∑X
i =1
ij = 1 for j = 1,2,….n (Arrive at a city once only)
Problem 5.14.
A salesman stationed at city A has to decide his tour plan to visit cities B, C, D, E and back to city
A I the order of his choice so that total distance traveled is minimum. No sub touring is permitted. He
cannot travel from city A to city A itself. The distance between cities in Kilometers is given below:
Cities A B C D E
A M 16 18 13 20
B 21 M 16 27 14
C 12 14 M 15 21
D 11 18 19 M 21
E 16 14 17 12 M
Instead of big M we can use infinity also. Or any element, which is sufficiently larger than all the
elements in the matrix, can be used.
Solution
COCM:
Cities A B C D E
A M 3 5 0 7
B 7 M 2 13 0
C 0 2 M 3 9
D 0 7 8 M 10
E 4 2 5 0 M
244 Operations Research
TOCM:
○
○
Cities A B C D E
○
A M 1 3 0 7
○
○
B 7 M 0 13 0
○
○
C 0 0 M 3 9
○
○
D 0 5 6 M 10
○
○
E 4 0 3 0 M
○
○
We can make only 4 assignments. Hence modify the matrix. Smallest element in the uncovered
cells is 3, deduct this from all other uncovered cells and add this to the elements at the crossed cells. Do
not alter the elements in cells covered by the line.
TOCM
○
○
Cities A B C D E
○
○
○ ○ ○ ○ ○ ○ ○ ○ ○ ○ ○ ○ ○ ○ ○ ○ ○ ○ ○ ○ ○ ○ ○ ○ ○ ○
○
○
A M 1 3 0 7
○
○
○
○
B 7 M 0 13 0
○
○
○
○
C 0 0 M 3 9
○
○
○
D 0 5 6 M 10
○
E 4 0 3 0 M
○
○
○
○
○
We can make only 4 assignments. Hence once again modify the matrix.
Sequencing: A to C, C to B, B to E, E to D, and D to A. As there is a tie
TOCM:
Cities A B C D E
A M 1 0 0 4
B 10 M 0 16 0
x
C 0x 0 M 3 6
D 0 5 3 M 7
E 4 0x 0 0 M
Problem 5.15.
Given the set up costs below, show how to sequence the production so as to minimize the total
setup cost per cycle.
Linear Programming - III Assignment Model 245
Jobs A B C D E
A M 2 5 7 1
B 6 M 3 8 2
C 8 7 M 4 7
D 12 4 6 M 5
E 1 3 2 8 M
Solution
COCM:
Jobs A B C D E
A M 1 4 6 0
B 4 M 1 6 0
C 4 3 M 0 3
D 8 0 2 M 1
E 0 2 1 7 M
TOCM:
Jobs A B C D E
A M 1 3 6 0
B 4 M 0 6 0x
C 4 3 M 0 3
D 8 0 1 M 1
E 0 2 0x 7 M
We can draw five lines and make assignment. The assignment is:
From A to E and From E to A cycling starts, which is not allowed in salesman problem. Hence
what we have to do is to select the next higher element than zero and make assignment with those
elements. After assignment of next higher element is over, then come to zero for assignment. If we
cannot finish the assignment with that higher element, then select next highest element and finish
assigning those elements and come to next lower element and then to zero. Like this we have to finish
all assignments. In this problem, the next highest element to zero is 1. Hence first assign all ones and
then consider zero for assignment. Now we shall first assign all ones and then come to zero.
TOCM:
Jobs A B C D E
A M 1 3 6 0x
B 4 M 0 6 0x
C 4 3 M 0 3
D 8 0 1 M 1
E 0 2 0x 7 M
246 Operations Research
Problem 5.16.
Solve the traveling salesman problem by using the data given below:
C12 = 20, C13 = 4, C14 = 10, C23 = 5, C34 = 6, C25 = 10, C35 = 6, C45 = 20 and Cij = Cji . And there
is no route between cities 'i' and 'j' if a value for Cij is not given in the statement of the problem. (i and
j are = 1,2,..5)
Solution
Cities 1 2 3 4 5
1 M 20 4 10 M
2 20 M 5 M 10
3 4 5 M 6 6
4 10 M 6 M 20
5 M 10 6 20 M
Now let us work out COCM/ROCM and TOCM, and then make the assignment.
TOCM:
Cities. 1 2 3 4 5
1 M 12 0 0x M
2 11 M 0x M 0
3 0x 1 M 0 1
4 0 M 0x M 9
5 M 0 0x 8 M
The sequencing is: 1 to3, 3 to 4, 4 to 1 and 1 to 3 etc., Cycling starts. Hence we shall start
assigning with 1 the next highest element and then assign zeros. Here also we will not get the sequencing.
Next we have to take the highest element 8 then assign 1 and then come to zeros.
TOCM:
Cities. 1 2 3 4 5
1 M 12 0 0 M
2 11 M 0 M 0
3 0 1 M 0 1
4 0 M 0 M 9
5 M 0 0 8 M
Problem 5.17.
A tourist organization is planning to arrange a tour to 5 historical places. Starting from the head
office at A then going round B, C, D and E and then come back to A. Their objective is to minimize the
total distance covered. Help them in sequencing the cities. A, B, C, D and E as the shown in the figure.
The numbers on the arrows show the distances in Km.
Solution
The distance matrix is as given below:
Places A B C D E
A M 20 M 10 10
B 20 M 30 M 35
C M 30 M 15 20
D 10 M 15 M 20
E 10 35 20 20 M
COCM
Places A B C D E
A M 10 M 0 0
B 0 M 10 M 15
C M 15 M 0 5
D 0 M 5 M 10
E 0 25 10 10 M
TOCM:
○
○
○
Places A B C D E
○
○
○
○
○
A M 0 M 0 0
○
○ ○ ○ ○ ○ ○ ○ ○ ○ ○ ○ ○ ○ ○ ○ ○ ○ ○ ○ ○ ○ ○ ○ ○ ○
○
○
○
○
B 0 M 5 M 15
○
○
○
○
○
C M 5 M 0 5
○
○
○
○
○
D 0 M 0 M 10
○
○
○
E 0 15 5 10 M
○
○
○
○
○
○
○
248 Operations Research
TOCM:
○
○
○
Places A B C D E
○
○
○ ○ ○ ○ ○ ○ ○ ○ ○ ○ ○ ○ ○ ○ ○ ○ ○ ○ ○ ○ ○ ○ ○ ○ ○ ○
A M 0 M 5 0
○
○
○
○
B 0 M 5 M 10
○
○
○
○
○ ○
C ○ ○ ○ ○ ○
M ○ ○ ○ ○
0 ○ ○ ○ ○
M ○ ○ ○ ○
0 ○ ○ ○
0 ○ ○ ○ ○
○
○
○
○
D 0 M 0 M 5
○
○
○
E 0 10 5 10 M
○
○
○
○
TOCM:
○
○
Places A ○
○
B C D E
○ ○ ○ ○ ○ ○ ○ ○ ○ ○ ○ ○ ○ ○ ○ ○ ○ ○ ○ ○ ○ ○ ○ ○ ○ ○
A M 0 M 5 0
○
○
B 0 M 5 M 5
○
○
○ ○ ○ ○ ○ ○ ○ ○ ○ ○ ○ ○ ○ ○ ○ ○ ○ ○ ○ ○ ○ ○ ○ ○ ○ ○ ○ ○
C M 0 M 0 0
○
○ ○ ○ ○ ○ ○ ○ ○ ○ ○ ○ ○ ○ ○ ○ ○ ○ ○ ○ ○ ○ ○ ○ ○ ○ ○ ○
○
D 0 M 0 M 0
○
E 0 5 5 5 M
○
○
○
Places A B C D E
A M 0 M 5 0x
B 0x M 0 M 0x
C M 0x M 0 0x
D 5 M 0x M 0
E 0 0x 0x 0x M
QUESTIONS
1. Four engineers are available to design four projects. Engineer 2 is not competent to design
the project B. Given the following time estimates needed by each engineer to design a given
project, find how should the engineers be assigned to projects so as to minimize the total
design time of four projects.
Engineers. Projects
A B C D
1 12 10 10 8
2 14 NOT 15 11
ELIGIBLE
3 6 10 16 4
4 8 10 9 7
2. (a) Explain the differences and similarities between Assignment problem and Transportation
problem.
(b) Explain why VAM or any other methods of getting basic feasible solution to a transportation
problem is not used to get a solution to assignment problem. What difficulties you come
across?
3. Explain briefly the procedure adopted in assignment algorithm.
4. Is traveling salesman problem is an assignment problem? If yes how? If not what are the
differences between assignment problem and traveling salesman problem.
5. What do you mean by balancing an assignment problem? What steps you take to solve
maximization case in assignment problem? Explain.
6. A Computer center has got three expert programmers. The center needs three application
programmes to be developed. The head of the computer center, after studying carefully the
programmes to be developed estimate the computer time in minutes required by the experts
to the application programmes as given in the matrix below. Assign the programmers to the
programmes in such a way that the total computer time is least.
Programmers. Programme.
A B C
1 120 100 80
2 70 90 110
3 110 140 120
7. (a). A company is faced with the problem of assigning six different machines to five different
jobs. The costs are estimated as follows in hundreds of rupees. Assign the jobs to machines
to minimize the total cost.